++
++
++
++
Which of the following statements regarding the epidemiology of cancer is true?
++
++
++
A. Cancer has equal mortality rates across all racial groups.
++
++
B. Cancer is responsible for one out of four deaths in the United States.
++
++
C. Cancer is the third leading cause of death in the United States.
++
++
D. Since 1992, the incidence of cancer has been increasing by about 2% each year.
++
++
E. The greatest number of cancers in the world occurs in Europe or North America.
+
++
The answer is B. (Chap. 99) Worldwide, there are more than 12.7 million new cases of cancer and 7.6 million cancer deaths each year, according to estimates provided by the International Agency for Research on Cancer. Most new cases of cancer occur in Asia (45%), with 26% occurring in Europe and 14.5% in North America. Worldwide, lung cancer is both the most common cause of cancer and the most common cause of cancer deaths. In the United States, lung cancer is the most common cause of cancer death but is not the most commonly diagnosed cancer. For men, the most commonly diagnosed cancer is prostate cancer, and for women, it is breast cancer. However, overall, in the United States, the incidence of cancer has been declining by about 2% each year since 1992. Despite the declining incidence, cancer is the second leading cause of death in the United States behind heart disease and is responsible for one out every four deaths. In individuals younger than 85 years, cancer is the leading cause of death. However, 5-year survival rates for cancer are generally improving over time. In 1960–1963, the 5-year survival for all cancers in white patients was 39%. By 2003–2009, this had increased to 69%. African American individuals with cancer fare more poorly with cancer. Over the same interval from 2003–2009, the 5- year survival was only 61% for black individuals. However, racial differences in survival are narrowing over time.
++
++
++
A 42-year-old woman is treated with carboplatin and paclitaxel for stage III ovarian cancer. Computed tomography (CT) imaging after completing six cycles of therapy shows that the tumor burden has decreased by 25%. What is the best assessment of her response to therapy?
++
++
++
++
++
++
++
++
++
+
++
The answer is D. (Chap. 99) Assessing response to treatment is an essential component of cancer treatment and compares repeat imaging to the imaging that was used to initially stage the disease. A complete response (option A) is defined as the disappearance of all evidence of disease. To qualify as a partial response (option B), an individual must experience at least a 50% reduction in tumor burden. This reduction is measured as the sum of the products of the perpendicular diameters of all measurable lesions. Another way that partial response can be measured is based upon a 30% decrease in the sum of the longest diameters of the lesions. Progressive disease (option C) is identified when there are any new lesions or if there has been a >25% increase in the sum of the products of the perpendicular diameters of all measurable lesions. The scenario proposed in the clinical history meets none of these criteria and, therefore, would be classified as stable disease (option D), which refers to tumor growth or shrinkage that fails to meet any of the definitions for response or progression.
++
++
++
A 68-year-old woman is diagnosed with stage II breast cancer. She has a history of severe chronic obstructive pulmonary disease with a forced expiratory volume in 1 second (FEV1) of 32% predicted, coronary artery disease with prior stenting of the left anterior descending artery, peripheral vascular disease, and obesity. She continues to smoke one to two packs of cigarettes every day. She requires oxygen at 2 L/min continuously and is functionally quite limited. She currently is able to attend to all of her activities of daily living, including showering and dressing. She retired from her work as a waitress 10 years previously due to her lung disease. At home, she does perform some of the household chores but is not able to use a vacuum. She does go out once or twice weekly to run typical errands and does drive. She feels short of breath with most of these activities and often uses a motorized cart when out and about. How would you categorize her performance status and prognosis for treatment taking this into consideration?
++
++
++
A. She has an Eastern Cooperative Oncology Group (ECOG) performance status of 1 and has a good prognosis with appropriate therapy.
++
++
B. She has an ECOG performance status of 2 and has a good prognosis with appropriate therapy.
++
++
C. She has an ECOG performance status of 3 and has a good prognosis with appropriate therapy.
++
++
D. She has an ECOG performance status of 3 and has a poor prognosis despite therapy.
++
++
E. She has an ECOG performance status of 4 and has a poor prognosis that precludes therapy.
+
++
The answer is B. (Chap. 99) While tumor burden is certainly a major factor in determining cancer outcomes, it is also important to consider the functional status of the patient when generating a therapeutic plan. The physiologic stresses of undergoing surgical interventions, radiation therapy, and chemotherapy can exhaust the limited reserves of a patient with multiple medical problems. It is clearly difficult to adequately measure the physiologic reserves of a patient, and most oncologists use performance status measures as a surrogate. Two of the most commonly used measures of performance status are the Eastern Cooperative Oncology Group (ECOG) and Karnofsky performance status. The ECOG scale provides a grade between 0 (fully active) and 5 (dead). Most patients are considered to have adequate reserve for undergoing treatment if the performance status is 0–2, with a status of 2 indicating someone who is ambulatory and capable of all self-care but unable carry out work activities. These individuals are up and about more than 50% of waking hours. A grade 3 performance score would indicate someone who is capable of only limited self-care and is confined to bed or chair more than 50% of waking hours. The Karnofsky score ranges from 0 (dead) to 100 (normal) and is graded at 10-point intervals. A Karnofsky score of <70 also indicates someone with poor performance status and would confer a worse prognosis.
++
++
++
Among women younger than 60 who die from cancer, which of the following is the most common organ of origin?
++
++
++
++
++
++
++
++
++
++
++
+
++
The answer is A. (Chap. 99) The cause of cancer death differs across the lifespan and between sexes. In both men and women who are under 20, the primary cause of cancer death is leukemia. In women between the ages of 20 and 59, breast cancer (option A) becomes the leading cause of cancer death. In men, leukemia remains the leading cause of cancer death until the age of 40. After age 40, lung cancer becomes the leading cause of cancer death in men, and in women, it becomes the leading cause of cancer death after age 60.
++
++
++
A 24-year-old woman is seen in follow-up 12 months after an allogeneic stem cell transplantation for acute myeloid leukemia. She is doing well without evidence of recurrent disease but has had manifestations of chronic graft-versus-host disease. She should be administered all of the following vaccines EXCEPT:
++
++
++
++
++
++
++
C. Measles, mumps, and rubella
++
++
D. Poliomyelitis via injection
++
++
E. 23-Valent pneumococcal polysaccharide
+
++
The answer is C. (Chap. 104) Patients who have undergone allogeneic stem cell transplantation remain at risk for infectious complications for an extended period despite engraftment and apparent return of normal hematopoietic capacity. Individuals with graft-versus-host disease (GVHD) often require immunosuppressive treatment that further increases infectious risk. Prevention of infection is the goal in these individuals, and the clinician should ensure appropriate vaccinations for all patients who have undergone intensive chemotherapy, have been treated for Hodgkin disease, or have undergone hematopoietic stem cell transplantation. In hematopoietic stem cell transplantations, the timeline for vaccination varies after transplantation. Pneumococcal vaccination (PCV13) can be given as early as 3–6 months after transplantation, but most vaccines are delayed until 6–12 months after transplantation. In general, the only vaccines that are given contain inactivated organisms. Therefore, oral vaccines for poliomyelitis and the varicella-zoster vaccine are contraindicated. The measles, mumps, and rubella vaccine is also a live virus vaccine, but can be safely given after 24 months if the patient does not have GVHD. Other recommended vaccines include diphtheria-tetanus, inactivated poliomyelitis (by injection), Haemophilus influenzae type B, hepatitis B, and 23-valent pneumococcal polysaccharide vaccine. Meningococcal vaccination is recommended in splenectomized patients and in those living in endemic areas, including college dormitories.
++
++
++
A 63-year-old man is treated with paclitaxel and carboplatin chemotherapy for stage IIIB adenocarcinoma of the lung. He presents for evaluation of a fever to 38.3°C (100.9°F). He is found to have erythema at the exit site of his tunneled catheter, although the tunnel itself is not tender or red. Blood cultures are negative at 48 hours. His neutrophil count is 1550/μL. What is the best approach to the management of this patient?
++
++
++
A. Removal of catheter alone
++
++
B. Treatment with ceftazidime and vancomycin
++
++
C. Treatment with topical antibiotics at the catheter site
++
++
D. Treatment with vancomycin alone
++
++
E. Treatment with vancomycin and removal of catheter
+
++
The answer is D. (Chap. 104) Clinicians are often faced with treatment decisions regarding catheter-related infections in patients who are immunocompromised from cancer and chemotherapy. Because many patients require several weeks of chemotherapy, tunneled catheters are often placed, and determining the need for catheter removal is an important consideration. When blood cultures are positive or there is evidence of infection along the track of the tunnel, catheter removal is recommended. When the erythema is limited to the exit site only, then it is not necessary to remove the catheter unless the erythema fails to respond to treatment. The recommended treatment for an exit site infection should be directed against coagulase-negative staphylococci. In the options presented, vancomycin alone is the best option for treatment. There is no need to add therapy for gram-negative organisms because the patient does not have neutropenia and has negative cultures.
++
++
++
A 44-year-old woman has myelodysplastic syndrome and has undergone myeloablative allogeneic stem cell transplantation. She has been neutropenic for 10 days and has developed a fever to 39.5°C (103.1°F). She has had a Port-a-Cath inserted for her intravenous access for the past 6 months. Her catheter site does not appear inflamed, and she has never tested positive for methicillin-resistant Staphylococcus aureus. What is the best initial choice of antibiotics for this patient?
++
++
++
++
++
++
++
C. Piperacillin-tazobactam
++
++
D. Any of the above would be an acceptable choice
++
++
E. Any of the above would be an acceptable choice with the addition of vancomycin
+
++
The answer is D. (Chap. 104) General guidelines for the treatment of febrile neutropenia depend on the expected duration of neutropenia, previous infections, and recent antibiotic exposures. Each febrile neutropenic patient should be approached as a unique problem. However, several general guidelines can help in treating these patients. The initial regimen should include antibiotics with activity against both gram-negative and gram-positive bacteria. If the expected duration of neutropenia is expected to be greater than 7 days, as in this scenario, then the initial antibiotic choice could be (1) ceftazidime or cefepime, (2) piperacillin/tazobactam, or (3) imipenem/cilastatin or meropenem; all of these regimens have shown equal efficacy in large clinical trials. These antibiotics exhibit broad-spectrum efficacy against gram-positive and gram-negative organisms, including Pseudomonas aeruginosa. Double coverage of P aeruginosa is not necessary, and use of aminoglycosides alone is contraindicated because these do not provide coverage of gram-positive organisms. Other antibiotics not providing adequate gram-positive coverage include aztreonam and fluoroquinolones. In addition, routine addition of vancomycin is also not indicated because studies have not shown improved outcomes with increased toxic effects. Vancomycin should only be added when there is high suspicion of coagulase-negative staphylococcal infection or specific concerns regarding methicillin-resistant Staphylococcus aureus infection. However, the treating physician needs to be knowledgeable about his or her local epidemiology and resistance patterns and prescribe in accordance with this knowledge. Antifungal therapy is often added when there is persistent fevers at 4–7 days without a known source of infection. The choice of specific antifungal agent (echinocandin, azole, lipid formulation of amphotericin B) would depend on whether the patient was receiving antifungal prophylaxis and whether there were reasons to suspect a specific source of infection, such as a pulmonary source.
++
++
++
A 42-year-old man is diagnosed with a stage I malignant melanoma on his left upper arm. Which of the following represents the strongest risk factor(s) for the development of this disease?
++
++
++
A. First-degree relative with melanoma
++
++
B. Light skin/hair/eye color
++
++
C. Number of total body nevi
++
++
++
++
+
++
The answer is E. (Chap. 105) Melanoma is an aggressive malignancy of the melanocytes that is seen predominantly in white-skinned individuals (~98% of cases) and has had a more than 17-fold increase in men and 9-fold increase in women in recent years. The strongest risk factors for the development of melanoma are the presence of multiple benign or atypical nevi and a family or personal history of the disease. Atypical nevi are often referred to as precursor lesions to melanoma, although the specific risk for any individual nevus is quite low. Only about 25% of melanomas arise from nevi; most arise de novo. Other risk factor for melanoma include presence of dysplastic nevi, ultraviolet exposure (including tanning beds), fair complexion, poor tanning ability, freckling, and specific genetic mutations, including CDKN2A, CDK4, and MITF.
++
++
++
A 55-year-old woman presents to her dermatologist with a lesion on her leg that is 8 mm in largest diameter and irregular in shape. She reports this mole has become larger and darker and wants to have it evaluated. Biopsy confirms melanoma that extends 0.5 mm from the surface and into the dermis with <1 mitosis/mm. Which of the following factors has the greatest impact on the patient’s prognosis?
++
++
++
++
++
++
++
++
++
++
++
+
++
The answer is B. (Chap. 105) The best predictor of metastatic risk in melanoma is Breslow thickness, which defines the absolute extent of tumor extension into the tissue. The Clark level defines the extent of invasion of the melanoma based on the layer of the skin involved but does not add significant prognostic information beyond the Breslow thickness. The number of mitoses is used in staging of tumors <1 mm in thickness to provide additional prognostic information about the likelihood of metastatic disease because patients with fewer mitoses have better long-term outcomes. Favorable anatomic sites for prognosis are the forearm and leg, with less favorable sites being the scalp, hands, feet, and mucous membranes. Women generally have better outcomes than men and are frequently diagnosed at earlier stages than men. The effect of age is not straightforward. Older patients are usually diagnosed with thicker primary tumors and have a later diagnosis, but younger patients have a greater risk of lymph node metastases.
++
++
++
A 46-year-old woman has previously had stage IIB melanoma removed from her upper back. She presents to the emergency department with dyspnea and is found to have multiple lung lesions concerning for metastatic disease. Prior to embarking upon chemotherapy for her disease, the presence of which genetic mutation is an indication for specific therapy?
++
++
++
++
++
++
++
++
++
++
++
+
++
The answer is A. (Chap. 105) Treatment of metastatic melanoma has largely shown little improvement in mortality in this disease, with median survival following diagnosis of metastatic disease of 6–15 months depending on organs involved. The prognosis is better for those with skin or subcutaneous metastases (M1a) than for those with lung metastases (M1b) and worst for those with liver, bone, or brain disease (M1c). Historically, no traditional chemotherapy regimen has had any effect on the outcome of metastatic melanoma, and these drugs are typically used only for symptom palliation. The agent with the most use in metastatic melanoma is interleukin-2 (IL-2). This cytokine requires individuals to be in good performance status, and the drug is often administered in an intensive care–like setting due to the high incidence of serious, but expected, side effects. IL-2 is not chemotherapy in the traditional sense, and the mechanism by which it kills tumor cells is not fully explained. It is thought to induce the activity of melanoma-specific T cells and leads to long-term disease-free survival in about 5% of treated patients. Other agents that alter the immune response to the tumor cells are the agents that cause immune checkpoint blockade. The only US Food and Drug Administration (FDA)–approved agent in this class is ipilimumab. This treatment is a monoclonal antibody that blocks CTLA-4 and results in improved T-cell function with eradication of tumor cells. This medication was the first treatment of any kind to show a survival benefit in metastatic melanoma. However, the response rate is only ~10%, and there is a significant side effect profile including induced autoimmunity that has limited the enthusiasm for the clinical use of the drug. In the past few years, two new classes of targeted therapies for melanoma have been introduced that have had fewer side effects, although the durability of response is unknown. It is now recommended that all newly diagnosed metastatic lesions undergo molecular testing for BRAF mutation. BRAF mutations are found in 40%–60% of melanomas and result in constitutive activation of the MAP kinase pathway. There are two currently approved BRAF inhibitors, vemurafenib and dabrafenib. These oral medications have shown tumor regression in approximately 50% of treated patients, although they are associated with a class-specific complication of the development of numerous skin lesions, which can include squamous cell cancer. An MEK inhibitor acts one step farther down the MAP kinase pathway and has also been approved by the FDA. Trametinib is less effective than the BRAF inhibitors as single-agent therapy but may improve survival when added to the BRAF inhibitors. Thus, the current recommended approach to the patient with metastatic melanoma is to test every patient for the presence of a “drugable” (BRAF) mutation. If the mutation is not present, the immunotherapy would be offered if the patient had an acceptable functional status. If the mutation is present, then the patient and physician would have to consider the pros and cons of the targeted therapy versus immunotherapy, as either could be acceptable. Targeted therapy has fewer side effects, but long-term durability of response is unknown. In contrast, immunotherapy has many more side effects and a lower initial response rate, but among responders, long-term durable responses can be achieved.
++
++
++
A 46-year-old woman has previously had stage IIB melanoma removed from her upper back. She presents to the emergency department with dyspnea and is found to have multiple lung lesions concerning for metastatic disease. You confirm the patient has the mutation of interest. Which of the following is recommended?
++
++
++
++
++
++
++
++
++
++
++
E. The specific therapy depends on the functional status and desires of the patient
+
++
The answer is E. (Chap. 105) Treatment of metastatic melanoma has largely shown little improvement in mortality in this disease, with median survival following diagnosis of metastatic disease of 6–15 months depending on organs involved. The prognosis is better for those with skin or subcutaneous metastases (M1a) than for those with lung metastases (M1b) and worst for those with liver, bone, or brain disease (M1c). Historically, no traditional chemotherapy regimen has had any effect on the outcome of metastatic melanoma, and these drugs are typically used only for symptom palliation. The agent with the most use in metastatic melanoma is interleukin-2 (IL-2). This cytokine requires individuals to be in good performance status, and the drug is often administered in an intensive care–like setting due to the high incidence of serious, but expected, side effects. IL-2 is not chemotherapy in the traditional sense, and the mechanism by which it kills tumor cells is not fully explained. It is thought to induce the activity of melanoma-specific T cells and leads to long-term disease-free survival in about 5% of treated patients. Other agents that alter the immune response to the tumor cells are the agents that cause immune checkpoint blockade. The only US Food and Drug Administration (FDA)–approved agent in this class is ipilimumab. This treatment is a monoclonal antibody that blocks CTLA-4 and results in improved T-cell function with eradication of tumor cells. This medication was the first treatment of any kind to show a survival benefit in metastatic melanoma. However, the response rate is only ~10%, and there is a significant side effect profile including induced autoimmunity that has limited the enthusiasm for the clinical use of the drug. In the past few years, two new classes of targeted therapies for melanoma have been introduced that have had fewer side effects, although the durability of response is unknown. It is now recommended that all newly diagnosed metastatic lesions undergo molecular testing for BRAF mutation. BRAF mutations are found in 40%–60% of melanomas and result in constitutive activation of the MAP kinase pathway. There are two currently approved BRAF inhibitors, vemurafenib and dabrafenib. These oral medications have shown tumor regression in approximately 50% of treated patients, although they are associated with a class-specific complication of the development of numerous skin lesions, which can include squamous cell cancer. An MEK inhibitor acts one step farther down the MAP kinase pathway and has also been approved by the FDA. Trametinib is less effective than the BRAF inhibitors as single-agent therapy but may improve survival when added to the BRAF inhibitors. Thus, the current recommended approach to the patient with metastatic melanoma is to test every patient for the presence of a “drugable” (BRAF) mutation. If the mutation is not present, the immunotherapy would be offered if the patient had an acceptable functional status. If the mutation is present, then the patient and physician would have to consider the pros and cons of the targeted therapy versus immunotherapy, as either could be acceptable. Targeted therapy has fewer side effects, but long-term durability of response is unknown. In contrast, immunotherapy has many more side effects and a lower initial response rate, but among responders, long-term durable responses can be achieved.
++
++
++
All of the following statements regarding nonmelanoma skin cancer are true EXCEPT:
++
++
++
A. Actinic keratoses and cheilitis are both premalignant forms of squamous cell carcinoma.
++
++
B. All forms of ultraviolet light exposure, including tanning beds, increase the risk of nonmelanoma skin cancers.
++
++
C. Basal cell carcinoma is most likely to become a metastatic malignancy.
++
++
D. Keratoacanthomas that regress spontaneously should be treated as aggressively as other squamous cell cancers as they progress to metastatic disease.
++
++
E. Solid organ transplantation is associated with a marked increased risk of both squamous and basal cell carcinoma that can be aggressive and lead to death.
+
++
The answer is C. (Chap. 105) Nonmelanoma skin cancer (NMSC) is the most common cancer in the United States, with an estimated annual incidence of 1.5–2 million cases yearly. However, most of these cases represent very limited disease with a low metastatic potential and account for only 2400 deaths yearly. The vast majority of NMSCs are basal cell carcinomas (BCC, 70%–80%) or squamous cell carcinomas (SCC, ~20%). The primary risk factor for all NMSCs is ultraviolet (UV) light exposure. UV exposure can occur either through direct exposure to sunlight (UVA and UVB exposure) or via tanning beds (97% UVA exposure). Other risk factors for NMSC include inherited disorders of nucleotide excision repair such as xeroderma pigmentosum, fair complexion, light hair/eyes, cigarette smoking, human immunodeficiency virus (HIV) infection, exposure to ionizing radiation, thermal burn scars, albinism, and chronic ulcerations. In addition, recipients of solid organ transplantations on chronic immunosuppression have a 65-fold increase in SCC and 10-fold increase in BCC. Moreover, NMSC in those with solid organ transplantation is more likely to be aggressive with higher rates of local recurrence, metastasis, and mortality. When comparing BCC to SCC, BCC is the less aggressive of the NMSCs and is typically a slowly enlarging, locally invasive neoplasm. The metastatic potential of BCC is <0.1%. SCC has a more variable natural history, depending on the lesion and host factors. Keratoacanthomas are rapidly growing, low-grade SCCs that can regress spontaneously without therapy. However, progression to metastatic disease has been reported after regression, so treatment for keratoacanthoma should be similar to that for other SCCs. Actinic keratoses and cheilitis are premalignant forms of SCC with transformation to malignancy occurring in 0.25%–20%. In general, the metastatic potential of SCC ranges from 0.3%–5.2%, with the greatest risk of metastases in tumors arising from non–sun-exposed tissues. The approach to treating NMSC depends on the size, depth, location, and host factors, with the primary goal being eradication of tumor with wide local margins.
++
++
++
A 65-year-old man presents to his primary care physician complaining of a hoarse voice for 6 months. He smokes one pack of cigarettes daily and also drinks at least a six pack of beer daily. His physical examination reveals a thin man with a weak voice in no distress. No stridor is heard. The head and neck examination is normal. No cervical lymphadenopathy is present. He is referred to an otolaryngologist who discovers a laryngeal lesion during flexible laryngoscopy. Biopsy reveals squamous cell carcinoma. On imaging, the mass measures 2.8 cm. No suspicious lymphadenopathy is present on positron emission tomography (PET) imaging. What is the best choice of therapy in this patient?
++
++
++
A. Concomitant chemotherapy and radiation therapy
++
++
++
++
C. Radiation therapy alone
++
++
D. Radical neck dissection alone
++
++
E. Radical neck dissection followed by concomitant chemotherapy and radiation
+
++
The answer is C. (Chap. 106) Cancers of the larynx often present with the subacute onset of hoarseness that does not resolve over time, but symptoms of head and neck cancer can be rather nonspecific. In more advanced cases, pain, stridor, dysphagia, odynophagia, and cranial neuropathies can occur. Diagnosis of head and neck cancer should include a computed tomography (CT) scan of the head and neck and endoscopic examination under anesthesia to perform biopsies. Positron emission tomography (PET) scans may be used as adjunctive therapy. The staging of head and neck cancers follows a tumor-node-metastasis (TNM) staging guideline (see Figure III-13). This patient would be staged as T2N0M0 based on tumor size without evidence of lymph node involvement or distant metastatic disease. With this designation, the patient’s overall stage would be stage II, and the disease would be classified as localized disease. The intent of therapy at this stage of disease is cure of cancer, and overall 5-year survival is 60%–90%. The choice of therapy for laryngeal cancer is radiation therapy to preserve voice. Surgical therapy could be chosen by the patient as well, but is less desirable. In locally or regionally advanced disease, patients can still be approached with curative intent, but this requires multimodality therapy with surgery followed by concomitant chemotherapy and radiation treatment.
+
+
++
FIGURE III-13 Tumor-node-metastasis (TNM) staging system.
+
++
++
++
All of the following have been identified as risk factors for the development of head and neck cancers EXCEPT:
++
++
++
++
++
B. Epstein Barr virus infection
++
++
C. Helicobacter pylori infection
++
++
D. Human papillomavirus infection
++
++
+
++
The answer is C. (Chap. 106) The number of new cases of head and neck cancers (oral cavity, pharynx, and larynx) in the United States was 53,640 in 2013, accounting for about 3% of adult malignancies; 11,520 people died from the disease. The worldwide incidence exceeds half a million cases annually. In North America and Europe, the tumors usually arise from the oral cavity, oropharynx, or larynx. The incidence of oropharyngeal cancers has been increasing in recent years. Nasopharyngeal cancer is more commonly seen in the Mediterranean countries and in the Far East, where it is endemic in some areas. Alcohol and tobacco use are the most significant risk factors for head and neck cancer, and when used together, they act synergistically. Smokeless tobacco is an etiologic agent for oral cancers. Other potential carcinogens include marijuana and occupational exposures such as nickel refining, exposure to textile fibers, and woodworking. Some head and neck cancers have a viral etiology. Epstein-Barr virus (EBV) infection is frequently associated with nasopharyngeal cancer, especially in endemic areas of the Mediterranean and Far East. In Western countries, the human papillomavirus (HPV) is associated with a rising incidence of tumors arising from the oropharynx (i.e., the tonsillar bed and base of tongue). Over 50% of oropharyngeal tumors are caused by HPV in the United States. HPV-16 is the dominant viral subtype, although HPV-18 and other oncogenic subtypes are seen as well. Alcohol- and tobacco-related cancers, on the other hand, have decreased in incidence. HPV-related oropharyngeal cancer occurs in a younger patient population and is associated with increased numbers of sexual partners and oral sexual practices. It is associated with a better prognosis, especially for nonsmokers.
++
++
++
Which of the following statements regarding a solitary pulmonary nodule is true?
++
++
++
A. A lobulated and irregular contour is more indicative of malignancy than a smooth one.
++
++
B. About 80% of incidentally found pulmonary nodules are benign.
++
++
C. Absence of growth over a period of 6–12 months is sufficient to determine whether a solitary pulmonary nodule is benign.
++
++
D. Ground glass nodules should be regarded as benign.
++
++
E. Multiple nodules indicate malignant disease.
+
++
The answer is A. (Chap. 107) A solitary pulmonary nodule is a frequent reason for referral to a pulmonologist, but most solitary pulmonary nodules are benign. In fact, over 90% of incidentally identified nodules are of benign origin. Features that are more likely to be present in a malignant lesion are size >3 cm, eccentric calcification, rapid doubling time, and lobulated and irregular contour. Ground glass appearance on CT imaging can be either malignant or benign. Among malignant lesions, ground glass infiltrate is seen more commonly in bronchoalveolar cell carcinoma. When multiple pulmonary nodules are identified, this most commonly represents prior granulomatous disease from healed infections. If multiple nodules are malignant in origin, this usually indicates disease metastatic to the lung, but can be simultaneous lung primary lesions or lesions metastatic from a primary lung cancer. Many incidentally identified nodules are too small to be diagnosed by biopsy and are nonspecific in nature. In this situation, it is prudent to follow the lesions for a period of 2 years, especially in a patient who is high risk for lung cancer, to allow for a proper doubling time to occur. If the lesion remains stable for 2 years, it is most likely benign, although some slow-growing tumors such bronchoalveolar cell carcinoma can have a slower growth rate.
++
++
++
A 64-year-old man seeks evaluation for a solitary pulmonary nodule that was found incidentally. He had presented to the emergency department for shortness of breath and chest pain. A CT pulmonary angiogram did not show any evidence of pulmonary embolism; however, a 9-mm nodule is seen in the periphery of the left lower lobe. No enlarged mediastinal lymph nodes are present. He is a current smoker of two packs of cigarettes daily and has done so since the age of 16. He generally reports no functional limitation related to respiratory symptoms. His FEV1 is 88% predicted, forced vital capacity is 92% predicted, and diffusion capacity is 80% predicted. He previously had a normal chest x-ray 3 years previously. What is the next best step in the evaluation and treatment of this patient?
++
++
++
A. Perform a bronchoscopy with biopsy for diagnosis
++
++
B. Perform a combined PET/CT to assess for uptake in the nodule and assess for lymph node metastases
++
++
C. Perform a follow-up CT scan in 3 months to assess for interval growth
++
++
D. Refer the patient to radiation oncology for stereotactic radiation of the dominant nodule
++
++
E. Refer the patient to thoracic surgery for video-assisted thoracoscopic biopsy and resection of lung nodule if malignancy is diagnosed
+
++
The answer is E. (Chap. 107) The evaluation and treatment of solitary pulmonary nodules are important to understand. This patient has a long smoking history with a new nodule that was not apparent by chest x-ray 3 years previously. This should be assumed to be a malignant nodule, and definitive diagnosis and treatment should be attempted. The option for diagnostic and staging procedures include PET/CT, bronchoscopic biopsy, percutaneous needle biopsy, or surgical biopsy with concomitant resection if positive. PET/CT would be low yield in this patient given the small size of the primary lesion (<1 cm) and the lack of enlarged mediastinal lymph nodes. Likewise, bronchoscopy would not provide a good yield because the lesion is very peripheral in origin, and a negative biopsy for malignancy would not be definitive. Appropriate approaches would be to either perform a percutaneous needle biopsy with CT guidance or perform a surgical biopsy with definitive resection if positive. Because this patient has preserved lung function, surgical biopsy and resection are good treatment options for this patient. A repeat CT scan assessing for interval growth would only be appropriate if the patient declined further workup at this time. Referral for treatment with radiation therapy is not appropriate in the absence of tissue diagnosis of malignancy, and surgical resection is the preferred primary treatment because the patient has no contraindications to surgical intervention.
++
++
++
A 62-year-old man presents to the emergency department complaining of a droopy right eye and blurred vision for the past day. The symptoms started abruptly, and he denies any antecedent illness. For the past 4 months, he has been complaining of increasing pain in his right arm and shoulder. His primary care physician has treated him for shoulder bursitis without relief. His past medical history is significant for chronic obstructive pulmonary disease and hypertension. He smokes one pack of cigarettes daily. He has chronic daily sputum production and stable dyspnea on exertion. On physical examination, he has right eye ptosis with unequal pupils. His pupil is 2 mm on the right and not reactive, whereas the pupil is 4 mm and reactive on the left. However, his ocular movements appear intact. His lung fields are clear to auscultation. On extremity examination, there is wasting of the intrinsic muscles of the hand. Which of the following would be most likely to explain the patient’s constellation of symptoms?
++
++
++
A. Enlarged mediastinal lymph nodes causing occlusion of the superior vena cava
++
++
B. Metastases to the midbrain from small-cell lung cancer
++
++
C. Paraneoplastic syndrome caused by antibodies to voltage-gated calcium channels
++
++
D. Presence of a cervical rib on chest x-ray
++
++
E. Right apical pleural thickening with a mass-like density measuring 1 cm in thickness
+
++
The answer is E. (Chap. 107) Pancoast syndrome results from apical extension of a lung mass into the brachial plexus with frequent involvement of the eighth cervical and first and second thoracic nerves. As the tumor continues to grow, it will also involve the sympathetic ganglia of the thoracic chain. The clinical manifestations of a Pancoast tumor include shoulder and arm pain and Horner syndrome (ipsilateral ptosis, miosis, and anhidrosis). Often, the shoulder and arm pain present several months prior to diagnosis. The most common cause of Pancoast syndrome is an apical lung tumor, usually non–small-cell lung cancer. Other causes include mesothelioma and infection, among others. Although midbrain lesions can cause Horner syndrome, other cranial nerve abnormalities would be expected.
++
Enlarged mediastinal lymph nodes and masses in the middle mediastinum can occlude the superior vena cava, leading to superior vena cava syndrome. Individuals with superior vena cava syndrome typically present with dyspnea and have evidence of facial and upper extremity swelling. Eaton-Lambert myasthenic syndrome is caused by antibodies to voltage-gated calcium channels and is characterized by generalized weakness of muscles that increases with repetitive nerve stimulation. Cervical ribs can cause thoracic outlet syndrome by compression of nerves or vasculature as they exit the chest. This typically presents with ischemic symptoms to the affected limb, but intrinsic wasting of the muscles of the hand can be seen due to neurologic compromise.
++
++
++
As an oncologist, you are considering treatment options for your patients with lung cancer, including small-molecule therapy targeting the epidermal growth factor receptor (EGFR). Which of the following patients is most likely to have an EGFR mutation?
++
++
++
A. A 23-year-old man with a hamartoma
++
++
B. A 33-year-old woman with a carcinoid tumor
++
++
C. A 45-year-old woman who has never smoked with an adenocarcinoma
++
++
D. A 56-year-old man with a 100-pack-year history of tobacco use with small-cell lung carcinoma
++
++
E. A 76-year-old man with squamous cell carcinoma and a history of asbestos exposure
+
++
The answer is C. (Chap. 107) Mutations of the epidermal growth factor receptor (EGFR) have recently been recognized as important mutations that affect the response of non–small-cell lung cancers to treatment with EGFR tyrosine kinase inhibitors. Initial studies of erlotinib in all patients with advanced non–small-cell lung cancer failed to show a treatment benefit; however, when only those patients with EGFR mutations were considered, treatment with anti-EGFR therapy improved progression-free and overall survival. Patients who are more likely to have EGFR mutations are women, nonsmokers, Asians, and patients with adenocarcinoma histopathology.
++
++
++
You are meeting today with Mr. Takei to discuss his recent diagnosis of small-cell lung cancer (SCLC). On reviewing his PET/CT results from earlier today, you note that he has a mass in the hilar region of his left lung and that he has a moderate pleural effusion there. You know that he underwent thoracentesis of that effusion last week, and so you call the pathologist to get a cytopathology report. He reports the presence of hyperchromic, small basophilic atypical cells in the pleural fluid, consistent with SCLC. Which of the following statements is true?
++
++
++
A. Thirty percent of patients with SCLC are diagnosed with the same stage of disease as Mr. Takei.
++
++
B. Mr. Takei has extensive-stage disease.
++
++
C. Surgical therapy alone has a high curative rate for Mr. Takei’s stage of SCLC.
++
++
D. The majority of patients with SCLC of this stage respond to chemotherapy alone and go into remission with a high 2-year survival.
++
++
E. Radiation plays no role in therapy for this disease.
+
++
The answer is B. (Chap. 107) The Veterans Administration staging system for small-cell lung cancer (SCLC) is a distinct two-stage system dividing patients into those with limited- or extensive-stage disease. Patients with limited-stage disease (LD) have cancer that is confined to the ipsilateral hemithorax and can be encompassed within a tolerable radiation port. Thus, contralateral supraclavicular nodes, recurrent laryngeal nerve involvement, and superior vena caval obstruction can all be part of LD. Patients with extensive-stage disease (ED) have overt metastatic disease by imaging or physical examination. Cardiac tamponade, malignant pleural effusion, and bilateral pulmonary parenchymal involvement generally qualify disease as ED, because the involved organs cannot be encompassed safely or effectively within a single radiation therapy port. Sixty to 70% of patients are diagnosed with ED at presentation. In general, surgical resection is not routinely recommended for patients because even patients with LD-SCLC still have occult micrometastases. Chemotherapy significantly prolongs survival in patients with SCLC. Despite response rates to first-line therapy as high as 80%, the median survival ranges from 12 to 20 months for patients with LD and from 7 to 11 months for patients with ED. Regardless of disease extent, the majority of patients relapse and develop chemotherapy-resistant disease. Only 6%–12% of patients with LD-SCLC and 2% of patients with ED-SCLC live beyond 5 years. The role of radiotherapy in ED-SCLC is largely restricted to palliation of tumor-related symptoms such as bone pain and bronchial obstruction.
++
++
++
Which of the following statements regarding screening for lung cancer in the National Lung Screening Trial using low-dose CT scanning is true?
++
++
++
A. Greater than 80% of positive results were found to be malignant after biopsy.
++
++
B. Positive results were found in approximately 5% of patients over the 3-year trial.
++
++
C. The trial compared the use of chest radiograph versus low-dose CT scan in patients 30–50 years old.
++
++
D. There was a reduction in lung cancer mortality in the low-dose CT group.
++
++
E. There was no difference in all-cause mortality between the CT and radiograph groups.
+
++
The answer is D. (Chap. 107 and N Engl J Med 2011;365:395–409) The National Lung Screening Trial (NLST), a randomized study designed to determine whether low-dose CT scan (LDCT) screening could reduce mortality from lung cancer in high-risk populations as compared with standard posterior-anterior chest x-ray (CXR), was published in 2011. High-risk patients were defined as individuals between 55 and 74 years of age with a ≥30 pack-year history of cigarette smoking; former smokers must have quit within the previous 15 years. Excluded from the trial were individuals with a previous lung cancer diagnosis, a history of hemoptysis, an unexplained weight loss of >15 lb in the preceding year, or a chest CT within 18 months of enrollment. A total of 53,454 persons were enrolled and randomized to annual screening yearly for 3 years (LDCT screening, n = 26,722; CXR screening, n = 26,732). Any noncalcified nodule measuring ≥4 mm in any diameter found on LDCT and CXR images with any noncalcified nodule or mass was classified as “positive.” Overall, 39.1% of participants in the LDCT group and 16% in the CXR group had at least one positive screening result. Of those who screened positive, the false-positive rate was 96.4% in the LDCT group and 94.5% in the CXR group. A greater number of cancers were found in the LDCT group. Nearly twice as many early stage IA cancers were detected in the LDCT group compared with the CXR group (40% vs. 21%). The overall rates of lung cancer death were 247 and 309 deaths per 100,000 participants in the LDCT and CXR groups, respectively, representing a 20% reduction in lung cancer mortality in the LDCT-screened population (95% confidence interval [CI], 6.8%–26.7%; p = .004). Compared with the CXR group, the rate of death in the LDCT group from any cause was reduced by 6.7% (95% CI, 1.2–13.6; p = .02). Despite the aforementioned caveats, screening of individuals who meet the NLST criteria for lung cancer risk seems warranted, provided comprehensive multidisciplinary coordinated care and follow-up similar to those provided to NLST participants are available.
++
++
++
++
A 34-year-old woman is seen by her internist for evaluation of right breast mass. This was noted approximately 1 week ago when she was showering. She has not had any nipple discharge or discomfort. She has no other medical problems. On examination, her right breast has a soft 1 cm × 2 cm mass in the right upper quadrant. There is no axillary lymphadenopathy present. The contralateral breast is normal. The breast is reexamined in 3 weeks, and the same findings are present. The cyst is aspirated, and clear fluid is removed. The mass is no longer palpable. Which of the following statements is true?
++
++
++
A. Breast magnetic resonance imaging (MRI) should be obtained to discern for residual fluid collection.
++
++
B. Mammography is required to further evaluate the lesion.
++
++
C. She should be evaluated in 1 month for recurrence.
++
++
D. She should be referred to a breast surgeon for resection.
++
++
E. She should not breastfeed any more children.
+
++
The answer is C. (Chap. 108) The patient has a breast cyst. This has a benign feel on examination, and aspiration of the mass showed nonbloody fluid with resolution of the mass. If there were residual mass or bloody fluid, mammogram and biopsy would be the next step. In patients such as this with nonbloody fluid in whom aspiration clears the mass, reexamination in 1 month is indicated. If the mass recurs, then aspiration should be repeated. If fluid recurs, mammography and biopsy would be indicated at that point. There is no indication at this point to refer for advanced imaging or surgical evaluation. Breastfeeding is not affected by breast cyst presence.
++
++
++
Which of the following women has the lowest risk of breast cancer?
++
++
++
A. A woman with menarche at 12 years, first child at 24 years, and menopause at 47 years
++
++
B. A woman with menarche at 14 years, first child at 17 years, and menopause at 52 years
++
++
C. A woman with menarche at 16 years, first child at 17 years, and menopause at 42 years
++
++
D. A woman with menarche at 16 years, first child at 32 years, and menopause at 52 years
++
++
+
++
The answer is C. (Chap. 108) Breast cancer risk is related to many factors, but age of menarche, age of first full-term pregnancy, and age and menopause together account for 70%–80% of all breast cancer risk. The lowest risk patients have the shortest duration of total menses (i.e., later menarche and earlier menopause), as well as early first full-term pregnancy. Specifically, the lowest risks are menarche at age 16 years old or older, first pregnancy by the age of 18 years, and menopause that begins 10 years before the median age of menopause of 52 years. Thus, patient C meets these criteria.
++
++
++
Which of the following history or physical examination findings should prompt investigation for hereditary nonpolyposis colon cancer screening in a 32-year-old man?
++
++
++
A. Father, paternal aunt, and paternal cousin with colon cancer with ages of diagnosis of 54, 68, and 37 years, respectively
++
++
B. Innumerable polyps visualized on routine colonoscopy
++
++
C. Mucocutaneous pigmentation
++
++
D. New diagnosis of ulcerative colitis
++
++
+
++
The answer is A. (Chap. 109) A strong family history of colon cancer should prompt consideration for hereditary nonpolyposis colon cancer (HNPCC), or Lynch syndrome, particularly if diffuse polyposis is not noted on colonoscopy. HNPCC is characterized by (1) three or more relatives with histologically proven colorectal cancer, one of whom is a first-degree relative and of the other two, at least one had the diagnosis before age 50; and (2) colorectal cancer in at least two generations. The disease is an autosomal dominant trait and is associated with other tumors, including in the endometrium and ovary. The proximal colon is most frequently involved, and cancer occurs with a median age of 50 years, 15 years earlier than in sporadic colon cancer. Patients with HNPCC are recommended to receive biennial colonoscopy and pelvic ultrasound beginning at age 25. Innumerable polyps suggest the presence of one of the autosomal dominant polyposis syndromes, many of which carry a high malignant potential. These include familial adenomatous polyposis, Gardner syndrome (associated with osteomas, fibromas, epidermoid cysts), or Turcot syndrome (associated with brain cancer). Peutz-Jeghers syndrome is associated with mucocutaneous pigmentation and hamartomas. Tumors may develop in the ovary, breast, pancreas, and endometrium; however, malignant colon cancers are not common. Ulcerative colitis is strongly associated with development of colon cancer, but it is unusual for colon cancer to be the presenting finding in ulcerative colitis. Patients are generally symptomatic from their inflammatory bowel disease long before cancer risk develops.
++
++
++
A 64-year-old woman presents with complaints of a change in stool caliber for the past 2 months. The stools now have a diameter of only the size of her fifth digit. Over this same period, she feels she has to exert increasing strain to have a bowel movement and sometimes has associated abdominal cramping. She often has blood on the toilet paper when she wipes. During this time, she has lost about 20 lb with a decreased appetite. On physical examination, the patient appears cachectic with a body mass index of 22.5 kg/m2. The abdomen is flat and nontender. The liver span is 12 cm to percussion. On digital rectal examination, a mass lesion is palpated approximately 6 cm into the rectum. A colonoscopy is attempted, which demonstrates a 2.5-cm sessile mass that narrows the distal colonic lumen. The biopsy confirms adenocarcinoma. The colonoscope is not able to traverse the mass. A CT scan of the abdomen does not show evidence of metastatic disease. Liver function tests are normal. A carcinoembryonic antigen level is 4.2 ng/mL. The patient is referred for surgery and undergoes rectosigmoidectomy with pelvic lymph node dissection. Final pathology demonstrates extension of the primary tumor into the muscularis propria, but not the serosa. Of 15 lymph nodes removed, 2 are positive for tumor. What do you recommend for this patient following surgery?
++
++
++
A. Chemotherapy with a regimen containing 5-fluorouracil
++
++
B. Complete colonoscopy within 3 months
++
++
C. Measurement of carcinoembryonic antigen levels at 3-month intervals
++
++
D. Radiation therapy to the pelvis
++
++
+
++
The answer is E. (Chap. 109) Colorectal cancer is the second most common cause of cancer death in the United States, and the mortality related to the disease has been decreasing in recent years. When colorectal cancer is identified, patients should be referred for surgical intervention because proper staging and prognosis cannot be determined without pathologic specimens if there is no gross evidence of metastatic disease. The preoperative workup to assess for metastatic or synchronous disease includes a complete colonoscopy if possible, chest radiograph, liver function testing, carcinoembryonic antigen (CEA) testing, and CT imaging of the abdomen. Staging of colorectal cancer follows a TNM staging system. However, the tumor staging is not based on absolute size of the tumor; rather, it is based on the extension of the tumor through the colonic wall. T1 tumors can extend into the submucosa but not beyond, T2 tumors extend into the muscularis propria, and T3 tumors involve the serosa and beyond. Nodal metastases are graded as N1 (one to three positive lymph nodes) and N2 (four or more positive lymph nodes). This patient’s stage of cancer would be T2N1M0 and would stage this as a stage III cancer. Despite the relatively advanced stage, the overall 5-year survival would be 50%–70% due to improvements in overall care of the patient with colorectal cancer. Since the patient has an occluding lesion that prevents preoperative colonoscopy, the patient needs to have a complete colonoscopy performed within the first several months following surgery and every 3 years thereafter. Serial measurements of CEA every 3 months have also been advocated by some specialists. Annual CT scanning may be performed for the first 3 years following resection, although the utility of the practice is debated. Radiation therapy to the pelvis is recommended for all patients with rectal cancer because it reduces local recurrence rate, especially in stage II and III tumors. When postoperative radiation therapy is combined with chemotherapeutic regimens containing 5-fluorouracil, the local recurrence rate is further reduced and overall survival is increased as well.
++
++
++
A 56-year-old man presents to a physician with weight loss and dysphagia. He feels that food gets stuck in his mid-chest such that he no longer is able to eat meats. He reports his diet consists primarily of soft foods and liquids. The symptoms have progressively worsened over 6 months. During this time, he has lost about 50 lb. He occasionally gets pain in his mid-chest that radiates to his back and also occasionally feels that he regurgitates undigested foods. He does not have a history of gastroesophageal reflux disease. He does not regularly seek medical care. He is known to have hypertension but takes no medications. He drinks 500 mL or more of whiskey daily and also smokes 1.5 packs of cigarettes per day. On physical examination, the patient appears cachectic with temporal wasting. He has a body mass index of 19.4 kg/m2. His blood pressure is 198/110 mmHg, heart rate is 110 bpm, respiratory rate is 18 breaths/min, temperature is 37.4°C (99.2°F), and oxygen saturation is 93% on room air. His pulmonary examination shows decreased breath sounds at the apices with scattered expiratory wheezes. His cardiovascular examination demonstrates an S4 gallop with a hyperdynamic precordium. A regular tachycardia is present. Blood pressures are equal in both arms. Liver span is not enlarged. There are no palpable abdominal masses. What is the most likely cause of the patient’s presentation?
++
++
++
A. Adenocarcinoma of the esophagus
++
++
B. Ascending aortic aneurysm
++
++
++
++
++
++
E. Squamous cell carcinoma of the esophagus
+
++
The answer is E. (Chap. 109) Esophageal cancer has a high mortality rate as most patients do not present until advanced disease is present. The typical presenting symptoms of esophageal cancer are dysphagia and significant weight loss. Dysphagia is typically fairly rapidly progressive over a period of weeks to months. Dysphagia initially is only to solid foods but progresses to include semisolids and liquids. For dysphagia to occur, an estimated 60% of the esophageal lumen must be occluded. Weight loss occurs due to decreased oral intake in addition to the cachexia that is common with cancer. Associated symptoms may include pain with swallowing that can radiate to the back, regurgitation or vomiting of undigested food, and aspiration pneumonia. The two major cell types of esophageal cancer in the United States are adenocarcinoma and squamous cell carcinoma, which have different risk factors. Individuals with squamous cell carcinomas typically have a history of both tobacco and alcohol abuse, whereas those with adenocarcinoma more often have a history of long-standing gastroesophageal reflux disease and Barrett esophagitis. Among those with a history of alcohol and tobacco abuse, there is an increased risk with increased intake, and interestingly, risk is more associated with whiskey drinking when compared to wine or beer. Other risk factors for squamous cell carcinoma of the esophagus include ingestion of nitrites, smoked opiates, fungal toxins in pickled vegetables, and physical insults that include long-standing ingestion of very hot tea or lye.
++
++
++
Which of the following risk factors is associated with both adenocarcinoma and squamous cell carcinoma of the esophagus?
++
++
++
++
++
B. Chronic gastroesophageal reflux disease
++
++
++
++
++
++
+
++
The answer is C. (Chap. 109) A variety of causative factors have been implicated in the development of squamous cell cancers of the esophagus. In the United States, the etiology of such cancers is primarily related to excess alcohol consumption and/or cigarette smoking. The relative risk increases with the amount of tobacco smoked or alcohol consumed, with these factors acting synergistically. Squamous cell esophageal carcinoma has also been associated with the ingestion of nitrates, smoked opiates, and fungal toxins in pickled vegetables, as well as mucosal damage caused by such physical insults as long-term exposure to extremely hot tea, the ingestion of lye, radiation-induced strictures, and chronic achalasia. The presence of an esophageal web in association with glossitis and iron deficiency (i.e., Plummer-Vinson or Paterson-Kelly syndrome) and congenital hyperkeratosis and pitting of the palms and soles (i.e., tylosis palmaris et plantaris) have each been linked with squamous cell esophageal cancer, as have dietary deficiencies of molybdenum, zinc, selenium, and vitamin A. Several strong etiologic associations have been observed to account for the development of adenocarcinoma of the esophagus. Such tumors arise in the distal esophagus in association with chronic gastric reflux, often in the presence of Barrett esophagus (replacement of the normal squamous epithelium of the distal esophagus by columnar mucosa), which occurs more commonly in obese individuals. Adenocarcinomas arise within dysplastic columnar epithelium in the distal esophagus. Cigarette smoking is associated with the development of adenocarcinoma of the esophagus as well.
++
++
++
All of the following conditions are known to increase the risk of developing hepatocellular carcinoma EXCEPT:
++
++
++
A. Cirrhosis from any cause
++
++
++
++
++
++
D. Nonalcoholic fatty liver
++
++
E. Nonalcoholic steatohepatitis
+
++
The answer is C. (Chap. 111) Hepatitis B virus (HBV) and hepatitis C virus (HCV) infection both have a clear relationship with the subsequent development of hepatocellular carcinoma (HCC). Both case-control and cohort studies have shown a strong association between chronic hepatitis B carrier rates and increased incidence of HCC. In Taiwanese male postal carriers who were hepatitis B surface antigen (HBsAg) positive, a 98-fold greater risk for HCC was found compared to HBsAg-negative individuals. HBV-based HCC may involve rounds of hepatic destruction with subsequent proliferation and not necessarily frank cirrhosis. The latency of HCV infection and development of HCC is approximately 30 years. Patients with HCV-associated HCC tend to have more frequent and advanced cirrhosis, but in HBV-associated HCC, only half the patients have cirrhosis, with the remainder having chronic active hepatitis. Approximately 75%–80% of patients with HCC have cirrhosis, and other liver conditions without cirrhosis, such as nonalcoholic fatty liver and nonalcoholic steatohepatitis, are associated with the development of HCC. Natural chemical carcinogens, such as aflatoxin B1, are strongly associated with HCC. It can be found in a variety of stored grains in hot, humid places, where peanuts and rice are stored in unrefrigerated conditions. Aflatoxin contamination of foodstuffs correlates well with incidence rates in Africa and to some extent in China. In endemic areas of China, even farm animals such as ducks have HCC. Malaria is not associated with HCC.
++
++
++
++
A 59-year-old man with known cirrhosis due to prior hepatitis C infection is brought to the clinic by his family due to complaints of 1 month of worsening malaise, abdominal bloating, and nausea with 1 week of right upper quadrant pain. His physical examination is notable for normal vital signs (baseline low blood pressure) and new hepatomegaly. Which of the following statements is true regarding the possibility of hepatocellular carcinoma?
++
++
++
A. Fluorodeoxyglucose (FDG)-PET scan is more sensitive for showing primary tumor than CT or ultrasound.
++
++
B. Hepatomegaly is an uncommon finding in hepatocellular carcinoma.
++
++
C. Imaging criteria alone (without biopsy) have a <75% specificity for the diagnosis.
++
++
D. Serum α-fetoprotein is the most sensitive test but is not specific.
++
++
E. Ultrasound is an excellent screening test for this patient.
+
++
The answer is E. (Chap. 111) Hepatomegaly is the most common physical sign in patients with HCC, occurring in 50%–90% of the patients. Abdominal bruits are noted in 6%–25%, and ascites occurs in 30%–60% of patients. α-Fetoprotein (AFP) is a serum tumor marker for HCC; however, it is only increased in approximately one-half of U.S. patients. Rising AFP in a patient at risk of HCC may be a marker of development of disease, and in some cases, serial measurement of AFP may be used as a marker of response to therapy. An ultrasound examination of the liver is an excellent screening tool. The two characteristic vascular abnormalities are hypervascularity of the tumor mass (neovascularization or abnormal tumor-feeding arterial vessels) and thrombosis by tumor invasion of otherwise normal portal veins. To determine tumor size and extent and the presence of portal vein invasion accurately, a helical/triphasic CT scan of the abdomen and pelvis, with fast-contrast bolus technique, should be performed to detect the vascular lesions typical of HCC. Portal vein invasion is normally detected as an obstruction and expansion of the vessel. Magnetic resonance imaging (MRI) can also provide detailed information, especially with the newer contrast agents. A prospective comparison of triphasic CT, gadolinium-enhanced MRI, ultrasound, and fluorodeoxyglucose (FDG)-PET showed similar results for CT, MRI, and ultrasound; PET imaging appears to be positive in only a subset of HCC patients. MRI is better able to distinguish dysplastic or regenerative nodules from HCC. Imaging criteria have been developed for HCC that do not require biopsy proof, as they have >90% specificity. The criteria include nodules >1 cm with arterial enhancement and portal venous washout and, for small tumors, specified growth rates on two scans performed less than 6 months apart (Organ Procurement and Transplant Network). Nevertheless, explant pathology after liver transplant for HCC has shown that approximately 20% of patients diagnosed without biopsy did not actually have a tumor.
++
++
++
A 59-year-old man with known cirrhosis due to prior hepatitis C infection is brought to the clinic by his family due to complaints of 1 month of worsening malaise, abdominal bloating, and nausea with 1 week of right upper quadrant pain. His physical examination is notable for normal vital signs (baseline low blood pressure) and new hepatomegaly. The patient described is found to have a 4-cm single hepatocellular carcinoma lesion. His performance status is excellent despite his recent decline. He still works as a web designer and walks over 10,000 steps daily. Based on this information, he may be eligible for all of the following therapies EXCEPT:
++
++
++
A. Cadaveric liver transplantation
++
++
B. Living donor liver transplantation
++
++
C. Local ablation with ethanol injection
++
++
D. Local radiofrequency ablation
++
++
+
++
The answer is E. (Chap. 111) Because this patient has a tumor >3 cm, he is not a candidate for primary radical resection. However, with his otherwise good prognosis and excellent performance status, he may be a candidate for liver transplantation that would cure his cirrhosis and his carcinoma. There are also evolving local ablative modalities that he would be eligible to receive (Figure III-29).
+
+
++
FIGURE III-29 Barcelona Clinic Liver Cancer (BCLC) staging classification and treatment schedule. Patients with very early hepatocellular carcinoma (HCC) (stage 0) are optimal candidates for resection. Patients with early HCC (stage A) are candidates for radical therapy (resection, liver transplantation [LT], or local ablation via percutaneous ethanol injection [PEI] or radiofrequency [RF] ablation). Patients with intermediate HCC (stage B) benefit from transcatheter arterial chemoembolization (TACE). Patients with advanced HCC, defined as presence of macroscopic vascular invasion, extrahepatic spread, or cancer-related symptoms (Eastern Cooperative Oncology Group performance status 1 or 2) (stage C), benefit from sorafenib. Patients with end-stage disease (stage D) will receive symptomatic treatment. Treatment strategy will transition from one stage to another on treatment failure or contraindications for the procedures. CLT, cadaveric liver transplantation; LDLT, living donor liver transplantation; PST, Performance Status Test. (Modified from JM Llovet et al: JNCI 100:698, 2008.)
+
++
++
++
All of the following statements regarding cholangiocarcinoma are true EXCEPT:
++
++
++
A. Asians infected with liver flukes have an increased risk of cholangiocarcinoma.
++
++
B. In eligible patients with cholangiocarcinoma, liver transplant plus radiation has a >60% 5-year recurrence-free survival.
++
++
C. Most patients present due to an abnormal screening ultrasound, without symptoms.
++
++
D. Primary biliary cirrhosis and hepatitis C virus infection are associated with cholangiocarcinoma.
++
++
E. The incidence of cholangiocarcinomas has been increasing in recent years.
+
++
The answer is C. (Chap. 111) Cholangiocarcinoma (CCC) typically refers to mucin-producing adenocarcinomas (different from HCC) that arise from the biliary tract and have features of cholangiocyte differentiation. They are grouped by their anatomic site of origin, as intrahepatic (IHC), perihilar (central, ~65% of CCCs), and peripheral (or distal, ~30% of CCCs). IHC is the second most common primary liver tumor. They arise as a result of cirrhosis less frequently than HCC, but may complicate primary biliary cirrhosis. However, cirrhosis and both primary biliary cirrhosis and HCV predispose to IHC. Nodular tumors arising at the bifurcation of the common bile duct are called Klatskin tumors and are often associated with a collapsed gallbladder, a finding that mandates visualization of the entire biliary tree. Incidence is increasing. Although most CCCs have no obvious cause, a number of predisposing factors have been identified, including primary sclerosing cholangitis (10%–20% of primary sclerosing cholangitis patients), an autoimmune disease, and liver fluke in Asians, especially Opisthorchis viverrini and Clonorchis sinensis. CCC also seems to be associated with any cause of chronic biliary inflammation and injury, including alcoholic liver disease, choledocholithiasis, choledochal cysts (10%), and Caroli disease (a rare inherited form of bile duct ectasia). CCC most typically presents as painless jaundice, often with pruritus or weight loss. Incidence has been increasing in recent decades; few patients survive 5 years. The usual treatment is surgical, but combination systemic chemotherapy may be effective. After complete surgical resection for IHC, 5-year survival is 25%–30%. Combination radiation therapy with liver transplantation has produced a 5-year recurrence-free survival rate of 65%.
++
++
++
All of the following statements regarding pancreatic cancer are true EXCEPT:
++
++
++
A. Alcohol consumption is not a risk factor for pancreatic cancer.
++
++
B. Cigarette smoking is a risk factor for pancreatic cancer.
++
++
C. Despite accounting for <5% of malignancies diagnosed in the United States, pancreatic cancer is the fourth leading cause of cancer death.
++
++
D. If detected early, the 5-year survival is up to 20%
++
++
E. The 5-year survival rates for pancreatic cancer have improved substantially in the last decade.
+
++
The answer is E. (Chap. 112) Pancreatic cancer is the fourth leading cause of cancer death in the United States, despite representing only 3% of all newly diagnosed malignancies. Infiltrating ductal adenocarcinomas account for the vast majority of cases and arise most frequently in the head of pancreas. At the time of diagnosis 85%–90% of patients have inoperable or metastatic disease, which is reflected in the 5-year survival rate of only 5% for all stages combined. An improved 5-year survival of up to 20% may be achieved when the tumor is detected at an early stage and when complete surgical resection is accomplished. Over the past 30 years, 5-year survival rates have not improved substantially. Cigarette smoking may be the cause of up to 20%–25% of all pancreatic cancers and is the most common environmental risk factor for this disease. Other risk factors are not well established due to inconsistent results from epidemiologic studies, but include chronic pancreatitis and diabetes. Alcohol does not appear to be a risk factor unless excess consumption gives rise to chronic pancreatitis.
++
++
++
A 65-year-old man is evaluated in clinic for 1 month of progressive painless jaundice and a 10-lb unintentional weight loss. His physical examination is unremarkable. A dual-phase contrast CT shows a suspicious mass in the head of the pancreas with biliary ductal dilation. Which of the following is the best diagnostic test to evaluate for suspected pancreatic cancer?
++
++
++
A. CT-guided percutaneous needle biopsy
++
++
B. Endoscopic ultrasound–guided needle biopsy
++
++
C. Endoscopic retrograde cholangiopancreatography with pancreatic juice sampling for cytopathology
++
++
++
++
+
++
The answer is B. (Chap. 112) Dual-phase, contrast-enhanced spiral CT is the imaging modality of choice to visualize suspected pancreatic masses. In addition to imaging the pancreas, it also provides accurate visualization of surrounding viscera, vessels, and lymph nodes. In most cases, this study can determine surgical resectability. There is no advantage of MRI over CT in predicting tumor resectability, but selected cases may benefit from MRI to characterize the nature of small indeterminate liver lesions and to evaluate the cause of biliary dilatation when no obvious mass is seen on CT. Preoperative confirmation of malignancy is not always necessary in patients with radiologic appearances consistent with operable pancreatic cancer. Endoscopic ultrasound–guided needle biopsy is the most effective technique to evaluate the mass for malignancy. It has an accuracy of approximately 90% and has a smaller risk of intraperitoneal dissemination compared with CT-guided percutaneous biopsy. Endoscopic retrograde cholangiopancreatography is a useful method for obtaining ductal brushings, but the diagnostic value of pancreatic juice sampling is only 25%–30%. CA 19-9 is elevated in approximately 70%–80% of patients with pancreatic carcinoma but is not recommended as a routine diagnostic or screening test because its sensitivity and specificity are inadequate for accurate diagnosis. Preoperative CA 19-9 levels correlate with tumor stage and prognosis. They are also an indicator of asymptomatic recurrence in patients with completely resected tumors. FDG-PET should be considered before surgery for detecting distant metastases.
++
++
++
A 63-year-old man presents to his internist with 3 months of worsening painless jaundice and anorexia. Further evaluation reveals a 1.5-cm obstructing lesion in the head of the pancreas that is confirmed as pancreatic adenocarcinoma by endoscopic ultrasound biopsy. The patient undergoes a modified Whipple procedure and is found to have a 1.6-cm primary tumor with no microscopic residual disease and negative lymph nodes. Which of the following statements regarding this patient is true?
++
++
++
A. He has a >75% expected 5-year survival after surgery.
++
++
B. He has pathologic stage II disease.
++
++
C. He should receive adjuvant chemotherapy.
++
++
D. The presence of SMAD4 gene inactivation in the tumor is a positive prognostic sign.
++
++
E. This presentation accounts for approximately 25% of patients with pancreatic cancer.
+
++
The answer is C. (Chap. 112) This patient has a <2-cm tumor with no microscopic residual disease and, therefore, has stage I disease, with a 20% 5-year survival after surgery and a gemcitabine-containing adjuvant chemotherapy regimen (Figure III-33). This presentation accounts for <10% of patients with newly diagnosed pancreatic cancer. Most patients present with advanced disease (stage IV), with a <5% 5-year survival. The four genes most commonly mutated or inactivated in pancreatic cancer are KRAS and the tumor-suppressor genes p16 (deleted in 95% of tumors), p53 (inactivated or mutated in 50%–70% of tumors), and SMAD4 (deleted in 55% of tumors). Typically, the cancer precursor lesions acquire these genetic abnormalities in a progressive manner that is associated with increasing dysplasia. SMAD4 gene inactivation is associated with a pattern of widespread metastatic disease in advanced-stage patients and poorer survival in patients with surgically resected pancreatic adenocarcinoma.
+
+
++
FIGURE III-33 AJCC, American Joint Committee on Cancer: TNM, tumor-node-metastasis. Illustration by Stephen Millward.
+
++
++
++
A 63-year-old man complains of notable pink-tinged urine for the last month. At first he thought it was due to eating beets, but it has not cleared. His medical history is notable for hypertension and cigarette smoking. He does report some worsening urinary frequency and hesitancy over the last 2 years. Physical examination is unremarkable. Urinalysis is notable for gross hematuria with no white cells or casts. Renal function is normal. Which of the following statements regarding this patient is true?
++
++
++
A. Cigarette smoking is not a risk for bladder cancer.
++
++
B. Gross hematuria makes prostate cancer more likely than bladder cancer.
++
++
C. If invasive bladder cancer with nodal involvement but no distant metastases is found, 5-year survival is 20%.
++
++
D. If superficial bladder cancer is found, intravesicular Bacillus Calmette-Guerin may be used as adjuvant therapy.
++
++
E. Radical cystectomy is generally recommended for invasive bladder cancer.
+
++
The answer is D. (Chap. 114) Bladder cancer is the 4th most common cancer in men and the 13th most common cancer in women. Cigarette smoking has a strong association with bladder cancer, particularly in men. The increased risk persists for at least 10 years after quitting. Bladder cancer is a small cause of cancer deaths because most detected cases are superficial with an excellent prognosis. Most cases of bladder cancer come to medical attention by the presence of gross hematuria emanating from exophytic lesions. Microscopic hematuria is more likely due to prostate cancer than bladder cancer. Cystoscopy under anesthesia is indicated to evaluate for bladder cancer. In cases of superficial disease, bacillus Calmette-Guerin is an effective adjuvant to decrease recurrence or treat unresectable superficial disease. In the United States, cystectomy is generally recommended for invasive disease. Even invasive cancer with nodal involvement has a >40% 10-year survival after surgery and adjuvant therapy.
++
++
++
A 68-year-old man comes to his physician complaining of 2 months of increasing right flank pain with 1 month of worsening hematuria. He was treated for cystitis at a walk-in clinic 3 weeks ago with no improvement. He also reports poor appetite and 5-lb weight loss. His physical examination is notable for a palpable mass in the right flank measuring >5 cm. His renal function is normal. All of the following are true about this patient’s likely diagnosis EXCEPT:
++
++
++
A. Anemia is more common than erythrocytosis.
++
++
B. Cigarette smoking increased his risk.
++
++
C. If his disease has metastasized, with best therapy, 5-year survival is >50%.
++
++
D. If his disease is confined to the kidney, 5-year survival is >80%.
++
++
E. The most likely pathology is clear cell carcinoma.
+
++
The answer is C. (Chap. 114) The incidence of renal cell carcinoma continues to rise and is now nearly 58,000 cases annually in the United States, resulting in 13,000 deaths. The male-to-female ratio is 2:1. Incidence peaks between the ages of 50 and 70 years, although this malignancy may be diagnosed at any age. Many environmental factors have been investigated as possible contributing causes; the strongest association is with cigarette smoking. Risk is also increased for patients who have acquired cystic disease of the kidney associated with end-stage renal disease and for those with tuberous sclerosis. Most renal cell carcinomas are clear cell tumors (60%), with papillary and chromophobic tumors being less common. Clear cell tumors account for >80% of patients that develop metastases. The classic triad of hematuria, flank pain, and palpable mass is only present in 10%–20% of cases initially. Most cases currently are found as incidental findings on CT scans or ultrasound obtained for different reasons. The increasing number of incidentally discovered low-stage tumors has contributed to an improved 5-year survival. The paraneoplastic phenomenon of erythrocytosis due to increased production of erythropoietin is only found in 3% of cases; anemia due to advanced disease is far more common. Stage I and II tumors are confined to the kidney and have a >80% survival after radical nephrectomy. Stage IV tumors with distant metastases have a 5-year survival of 10%. Renal cell carcinoma is notably resistant to traditional chemotherapeutic agents. Until recently, cytokine therapy with IL-2 or interferon-γ was used to produce regression in 10%–20% of patients with metastatic disease. Antiangiogenic medications have changed the treatment of advance renal cell carcinoma. Sunitinib was demonstrated to be superior to interferon-γ, and it is now first-line therapy for patients with advanced metastatic disease. Pazopanib and axitinib are newer agents of the same class as sunitinib. Pazopanib was compared to sunitinib in a randomized first-line phase III trial. Efficacy was similar, and there was less fatigue and skin toxicity, resulting in better quality of life scores for pazopanib compared with sunitinib. Temsirolimus and everolimus, inhibitors of the mammalian target of rapamycin (mTOR), show activity in patients with untreated poor-prognosis tumors and in sunitinib/sorafenib-refractory tumors. Patients may benefit from the sequential use of axitinib and everolimus following progression after first-line therapy. The prognosis of metastatic renal cell carcinoma is variable.
++
++
++
A 68-year-old man comes to his physician complaining of 2 months of increasing right flank pain with 1 month of worsening hematuria. He was treated for cystitis at a walk-in clinic 3 weeks ago with no improvement. He also reports poor appetite and 5-lb weight loss. His physical examination is notable for a palpable mass in the right flank measuring >5 cm. His renal function is normal. In the patient described, imaging shows a 10-cm solid mass in the right kidney and multiple nodules in the lungs consistent with metastatic disease. Needle biopsy of a lung lesion confirms the diagnosis of renal cell carcinoma. Which of the following is recommended therapy?
++
++
++
++
++
++
++
++
++
++
++
+
++
The answer is E. (Chap. 114) The incidence of renal cell carcinoma continues to rise and is now nearly 58,000 cases annually in the United States, resulting in 13,000 deaths. The male-to-female ratio is 2:1. Incidence peaks between the ages of 50 and 70 years, although this malignancy may be diagnosed at any age. Many environmental factors have been investigated as possible contributing causes; the strongest association is with cigarette smoking. Risk is also increased for patients who have acquired cystic disease of the kidney associated with end-stage renal disease and for those with tuberous sclerosis. Most renal cell carcinomas are clear cell tumors (60%), with papillary and chromophobic tumors being less common. Clear cell tumors account for >80% of patients that develop metastases. The classic triad of hematuria, flank pain, and palpable mass is only present in 10%–20% of cases initially. Most cases currently are found as incidental findings on CT scans or ultrasound obtained for different reasons. The increasing number of incidentally discovered low-stage tumors has contributed to an improved 5-year survival. The paraneoplastic phenomenon of erythrocytosis due to increased production of erythropoietin is only found in 3% of cases; anemia due to advanced disease is far more common. Stage I and II tumors are confined to the kidney and have a >80% survival after radical nephrectomy. Stage IV tumors with distant metastases have a 5-year survival of 10%. Renal cell carcinoma is notably resistant to traditional chemotherapeutic agents. Until recently, cytokine therapy with IL-2 or interferon-γ was used to produce regression in 10%–20% of patients with metastatic disease. Antiangiogenic medications have changed the treatment of advance renal cell carcinoma. Sunitinib was demonstrated to be superior to interferon-γ, and it is now first-line therapy for patients with advanced metastatic disease. Pazopanib and axitinib are newer agents of the same class as sunitinib. Pazopanib was compared to sunitinib in a randomized first-line phase III trial. Efficacy was similar, and there was less fatigue and skin toxicity, resulting in better quality of life scores for pazopanib compared with sunitinib. Temsirolimus and everolimus, inhibitors of the mammalian target of rapamycin (mTOR), show activity in patients with untreated poor-prognosis tumors and in sunitinib/sorafenib-refractory tumors. Patients may benefit from the sequential use of axitinib and everolimus following progression after first-line therapy. The prognosis of metastatic renal cell carcinoma is variable.
++
++
++
Which of the following has been shown in randomized trials to reduce the future risk of prostate cancer?
++
++
++
++
++
++
++
++
++
++
++
+
++
The answer is A. (Chap. 115) The results from several large, double-blind, randomized chemoprevention trials have established 5α-reductase inhibitors as the predominant therapy to reduce the future risk of a prostate cancer diagnosis. Randomized placebo-controlled trials have shown that finasteride and dutasteride reduce the period prevalence of prostate cancer. Trials of selenium, vitamin C, and vitamin E have shown no benefit versus placebo.
++
++
++
A 54-year-old man is evaluated in an executive health program. On physical examination, he is noted to have an enlarged prostate with a right lobe nodule. He does not recall his last digital rectal examination and has never had prostate-specific antigen (PSA) tested. Based on this evaluation, which of the following is recommended next?
++
++
++
A. Bone scan to evaluate for metastasis
++
++
++
++
C. PSA now and in 3 months to measure PSA velocity
++
++
D. Repeat digital rectal examination in 3 months
++
++
E. Transrectal ultrasound-guided biopsy
+
++
The answer is E. (Chap. 115) Transrectal ultrasound-guided biopsy is recommended for men with an abnormal digital rectal examination. Carcinomas are characteristically hard, nodular, and irregular, whereas induration may also be due to benign prostatic hypertrophy (BPH) or calculi. Overall, 20%–25% of men with an abnormal digital rectal examination have cancer. A diagnosis of cancer is established by an image-guided needle biopsy. Direct visualization by transrectal ultrasound or MRI assures that all areas of the gland are sampled. Contemporary schemas advise an extended pattern 12-core biopsy that includes sampling from the peripheral zone as well as a lesion-directed palpable nodule or suspicious image-guided sampling. Men with an abnormal prostate-specific antigen (PSA) level and negative biopsy are advised to undergo a repeat biopsy. When prostate cancer is diagnosed, a measure of histologic aggressiveness is assigned using the Gleason grading system, in which the dominant and secondary glandular histologic patterns are scored from 1 (well differentiated) to 5 (undifferentiated) and summed to give a total score of 2–10 for each tumor. The most poorly differentiated area of tumor (i.e., the area with the highest histologic grade) often determines biologic behavior. The presence or absence of perineural invasion and extracapsular spread is also recorded.
++
++
++
Which of the following statements regarding use of PSA is true?
++
++
++
A. Asymptomatic men with an elevated PSA should receive a 2-week course of antibiotics before repeating PSA and considering biopsy.
++
++
B. Most prostate cancer deaths occur in men with PSA levels below the top quartile.
++
++
C. PSA is produced by malignant and nonmalignant prostate cells.
++
++
D. The American Urological Association recommends PSA screening in men 40–55 years of age.
++
++
E. The US Preventive Services Task Force recommends PSA screening in men between 55 and 69 years of age.
+
++
The answer is C. (Chap. 115) PSA is a serine protease that causes liquefaction of seminal coagulum. It is produced by both nonmalignant and malignant epithelial cells and, as such, is prostate specific, not prostate cancer specific. Serum levels may also increase from prostatitis and benign prostatic hypertrophy. Serum levels are not significantly affected by digital rectal examination, but the performance of a prostate biopsy can increase PSA levels up to 10-fold for 8–10 weeks. PSA testing was approved by the U.S. FDA in 1994 for early detection of prostate cancer, and the widespread use of the test has played a significant role in the proportion of men diagnosed with early-stage cancers: more than 70%–80% of newly diagnosed cancers are clinically organ confined. The level of PSA in blood is strongly associated with the risk and outcome of prostate cancer. Most prostate cancer deaths (90%) occur among men with PSA levels in the top quartile (>2 ng/mL), although only a minority of men with PSA >2 ng/mL will develop lethal prostate cancer. Despite this and mortality rate reductions reported from large randomized prostate cancer screening trials, routine use of the test remains controversial. The U.S. Preventive Services Task Force (USPSTF) recently made a clear recommendation against screening. By giving a grade of D in the recommendation statement that was based on this review, the USPSTF concluded that “there is moderate or high certainty that this service has no net benefit or that the harms outweigh the benefits.” Whether the harms of screening, overdiagnosis, and overtreatment are justified by the benefits in terms of reduced prostate cancer mortality is open to reasonable doubt. In response to the USPSTF, the American Urological Association (AUA) updated their consensus statement regarding prostate cancer screening. They concluded that the quality of evidence for the benefits of screening was moderate, and evidence for harm was high for men age 55–69 years. For men outside this age range, evidence was lacking for benefit, but the harms of screening, including overdiagnosis and overtreatment, remained. The AUA recommends shared decision making considering PSA-based screening for men age 55–69, a target age group for whom benefits may outweigh harms. Outside this age range, PSA-based screening as a routine test was not recommended based on the available evidence. The PSA criteria used to recommend a diagnostic prostate biopsy have evolved over time. However, based on the commonly used cut point for prostate biopsy (a total PSA ≥4 ng/mL), most men with a PSA elevation do not have histologic evidence of prostate cancer at biopsy. In addition, many men with PSA levels below this cut point harbor cancer cells in their prostate. There is no PSA below which the risk of prostate cancer is zero. The routine use of antibiotics in an asymptomatic man with an elevated PSA level is strongly discouraged and should not affect the decision to pursue further evaluation.
++
++
++
All of the following medications may be useful in the treatment of benign prostatic hypertrophy EXCEPT:
++
++
++
++
++
++
++
++
++
++
++
+
++
The answer is B. (Chap. 115) Benign prostatic hypertrophy (BPH) is a pathologic process that contributes to the development of lower urinary tract symptoms in men. Such symptoms, arising from lower urinary tract dysfunction, are further subdivided into obstructive symptoms (urinary hesitancy, straining, weak stream, terminal dribbling, prolonged voiding, incomplete emptying) and irritative symptoms (urinary frequency, urgency, nocturia, urge incontinence, small voided volumes). Lower urinary tract symptoms and other sequelae of BPH are not just due to a mass effect, but are also likely due to a combination of the prostatic enlargement and age-related detrusor dysfunction. Asymptomatic patients do not require treatment regardless of the size of the prostate gland. Symptomatic relief is the most common reason men seek treatment for BPH, and therefore, the goal of therapy for BPH is usually relief of these symptoms. Selective α-adrenergic receptor antagonists, such as alfuzosin, are thought to treat the dynamic aspect of BPH by reducing sympathetic tone of the bladder outlet, thereby decreasing resistance and improving urinary flow. 5α-Reductase inhibitors, such as dutasteride and finasteride, are thought to treat the static aspect of BPH by reducing prostate volume and having a similar, albeit delayed effect. They have also proven to be beneficial in the prevention of BPH progression, as measured by prostate volume, the risk of developing acute urinary retention, and the risk of having BPH-related surgery. The use of an α-adrenergic receptor antagonist and a 5α-reductase inhibitor as combination therapy seeks to provide symptomatic relief while preventing progression of BPH.
++
Another class of medications that has shown improvement in lower urinary tract symptoms secondary to BPH is phosphodiesterase-5 (PDE5) inhibitors, used currently in the treatment of erectile dysfunction. All three of the PDE5 inhibitors available in the United States, sildenafil, vardenafil, and tadalafil, appear to be effective in the treatment of symptoms secondary to BPH. The use of PDE5 inhibitors is not without controversy, however, given the fact that short-acting PDE5 inhibitors such as sildenafil need to be dosed separately from α-blockers because of potential hypotensive effects. Bosentan is a nonselective endothelin receptor antagonist used in the treatment of pulmonary arterial hypertension.
++
++
++
A 26-year-old man presents with pain and swelling of his right testicle that has persisted after an empiric treatment for epididymitis. Ultrasound confirms a 1.5 × 2 cm solid mass, suspicious for testicular cancer. Radical inguinal orchiectomy confirms the mass as a seminoma with disease limited to the testis (tumor stage pT1). Chest, abdomen, and pelvis CT shows no evidence of metastatic disease or lymphadenopathy. Results of serum tumor markers demonstrate the following (normal values in parentheses): α-fetoprotein (AFP) 5 ng/mL (<10 ng/mL), β-human chorionic gonadotropin (β-hCG) 182 U/L (0.2–0.8 U/L), and lactate dehydrogenase (LDH) 432 U/L (100–190 U/L). Following resection, all tumor markers become undetectable after an appropriate interval. Which is the next best step in this patient’s treatment?
++
++
++
A. Immediate retroperitoneal radiation therapy
++
++
B. Nerve-sparing retroperitoneal lymph node dissection
++
++
C. Single-dose therapy with cisplatin
++
++
D. Surveillance alone with treatment only if relapse detected
++
++
E. Either A or D, because both are associated with a near 100% cure rate
+
++
The answer is E. (Chap. 116) Pure seminomas have the best survival of all forms of testicular cancer and represent approximately 50% of all germ cell tumors. The median age of presentation is the fourth decade of life, and approximately 80% of individuals present with stage I disease, indicating any disease limited to the testis no matter the size at initial presentation. All men presenting with a testicular mass should be referred for radical inguinal orchiectomy as this approach mirrors the embryonic development of the testis and does not breach anatomic barriers to allow for other pathways of spread. In the staging workup of testicular cancers, men should undergo CT imaging of the chest, abdomen, and pelvis as well as measurement of the serum tumor markers AFP and β-human chorionic gonadotropin (β-hCG), in addition to lactate dehydrogenase (LDH) levels. These tumor markers assist with both diagnosis and prognosis in testicular cancer and help with determining the appropriate postorchiectomy treatment. In pure seminomas, AFP levels should not be elevated. If the AFP level is elevated, this indicates an occult nonseminomatous component, which may require more aggressive initial treatment with either retroperitoneal lymph node dissection or adjuvant chemotherapy depending on local surgical expertise and preference of the patient and treating physician. β-hCG levels may be elevated in pure seminomas, although this too is infrequent in men without advanced disease. LDH levels are less specific, but are increased in up to 80% of patients with advanced seminoma. After resection, the tumor markers should return to normal values within their expected half-lives following first-order kinetics. The half-life of β-hCG is 24–36 hours, and that of AFP is 5–7 days. In stage I seminoma, survival is near 100% with either immediate postorchiectomy radiation or with surveillance alone (option E). Given the concern about secondary malignancy due to radiation exposure, many providers chose watchful waiting with surveillance alone in men who are compliant with follow-up. However, approximately 15% of patients will experience relapse, and 5% of relapses occur after 5 years. Thus, extended follow-up is required. A single dose of carboplatin has been investigated as an alternative to radiation therapy, but long-term outcomes are as yet unknown.
++
++
++
Which of the following statements describes the relationship between testicular tumors and serum markers?
++
++
++
A. β-hCG and AFP should be measured when following the progress of a tumor.
++
++
B. β-hCG is limited in its usefulness as a marker because it is identical to human luteinizing hormone.
++
++
C. Measurement of tumor markers the day after surgery for localized disease is useful in determining completeness of the resection.
++
++
D. More than 40% of nonseminomatous germ cell tumors produce no cell markers.
++
++
E. Pure seminomas produce AFP or β-hCG in more than 90% of cases.
+
++
The answer is A. (Chap. 116) Ninety percent of persons with nonseminomatous germ cell tumors produce either AFP or β-hCG; in contrast, persons with pure seminomas usually produce neither. These tumor markers are present for some time after surgery; if the presurgical levels are high, 30 days or more may be required before meaningful postsurgical levels can be obtained. The half-lives of AFP and β-hCG are 6 days and 1 day, respectively. After treatment, unequal reduction of β-hCG and AFP may occur, suggesting that the two markers are synthesized by heterogeneous clones of cells within the tumor; thus, both markers should be followed. β-hCG is similar to luteinizing hormone except for its distinctive β subunit.
++
++
++
All of the following statements regarding the risk of ovarian cancer are true EXCEPT:
++
++
++
A. Ten percent of women with ovarian cancer have a germline mutation in either BRACA1 or BRACA2.
++
++
B. Early prophylactic oophorectomy in women with BRACA1 or BRACA2 mutations reduces the risk of developing subsequent breast cancer.
++
++
C. Individuals with a single copy of a BRACA1 or BRACA2 mutant allele have an increased risk of breast and ovarian cancer.
++
++
D. Women with a mutation in BRACA1 have a higher risk of ovarian cancer than woman with a mutation in BRACA2.
++
++
E. Women with BRACA1, BRACA2, or other at-risk germline mutations should be screened with serial measurement of the CA-125 tumor marker.
+
++
The answer is E. (Chap. 117) A variety of genetic syndromes substantially increase a woman’s risk of developing ovarian cancer. Approximately 10% of women with ovarian cancer have a germline mutation in one of two DNA repair genes: BRCA1 (chromosome 17q12-21) or BRCA2 (chromosome 13q12-13). Individuals inheriting a single copy of a mutant allele have a very high incidence of breast and ovarian cancer. Most of these women have a family history that is notable for multiple cases of breast and/or ovarian cancer, although inheritance through male members of the family can camouflage this genotype through several generations. The most common malignancy in these women is breast carcinoma, although women harboring germline BRCA1 mutations have a marked increased risk of developing ovarian malignancies in their 40s and 50s with a 30%–50% lifetime risk of developing ovarian cancer. Women harboring a mutation in BRCA2 have a lower penetrance of ovarian cancer with perhaps a 20%–40% chance of developing this malignancy, with onset typically in their 50s or 60s. Women with a BRCA2 mutation also are at slightly increased risk of pancreatic cancer. Likewise, women with mutations in the DNA mismatch repair genes associated with Lynch syndrome type 2 (MSH2, MLH1, MLH6, PMS1, PMS2) may have a risk of ovarian cancer as high as 1% per year in their 40s and 50s. Finally, a small group of women with familial ovarian cancer may have mutations in other BRCA-associated genes such as RAD51, CHK2, and others. Screening studies in this select population suggest that current screening techniques, including serial evaluation of the CA-125 tumor marker and ultrasound, are insufficient at detecting early-stage and curable disease, so women with these germline mutations are advised to undergo prophylactic removal of ovaries and fallopian tubes typically after completing childbearing and ideally before age 35–40 years. Early prophylactic oophorectomy also protects these women from subsequent breast cancer, with a reduction of breast cancer risk of approximately 50%.
++
++
++
A 42-year-old woman seeks evaluation for over 6 months of postcoital bleeding without dyspareunia. She also notes some recent spotting between her regular menses. She has no past medical history, is unmarried with multiple sexual partners and practicing unprotected sex, and works as an accountant. She has not sought gynecologic evaluation for over 10 years. Pelvic examination reveals an abnormal appearance of the cervix with an abnormal Pap smear, positive human papillomavirus (HPV) test, and negative human immunodeficiency virus (HIV), chlamydia, gonorrhea, and syphilis studies. A cervical biopsy shows squamous cell carcinoma confined to the cervix. All of the following statements regarding this woman’s condition are true EXCEPT:
++
++
++
A. Cervical cancer is an uncommon cancer worldwide.
++
++
B. Her cancer is related to HPV infection.
++
++
C. HPV vaccination before initiation of sexual activity can decrease the risk of developing an abnormal Pap smear.
++
++
D. She has stage I cervical cancer.
++
++
E. With surgical therapy, her 5-year survival is >80%.
+
++
The answer is A. (Chap. 117) Cervical cancer is the second most common and most lethal malignancy in women worldwide likely due to the widespread infection with high-risk strains of HPV and limited utilization of or access to Pap smear screening in many nations throughout the world. Nearly 500,000 cases of cervical cancer are expected worldwide, with approximately 240,000 deaths annually. Cancer incidence is particularly high in women residing in Central and South America, the Caribbean, and southern and eastern Africa. Mortality rate is disproportionately high in Africa. In the United States, 12,360 women were diagnosed with cervical cancer, and 4020 women died of the disease in 2014. HPV is the primary neoplastic-initiating event in the vast majority of women with invasive cervical cancer. This double-strand DNA virus infects epithelium near the transformation zone of the cervix. More than 60 types of HPV are known, with approximately 20 types having the ability to generate high-grade dysplasia and malignancy. HPV-16 and -18 are the types most frequently associated with high-grade dysplasia and targeted by both U.S. FDA–approved vaccines. The large majority of sexually active adults are exposed to HPV, and most women clear the infection without specific intervention. Risk factors for HPV infection and, in particular, dysplasia include a high number of sexual partners, early age of first intercourse, and history of venereal disease. Smoking is a cofactor; heavy smokers have a higher risk of dysplasia with HPV infection. HIV infection, especially when associated with low CD4+ T-cell counts, is associated with a higher rate of high-grade dysplasia and likely a shorter latency period between infection and invasive disease. The administration of highly active antiretroviral therapy reduces the risk of high-grade dysplasia associated with HPV infection. Currently approved vaccines include the recombinant proteins to the late proteins, L1 and L2, of HPV-16 and -18. Vaccination of women before the initiation of sexual activity dramatically reduces the rate of HPV-16 and -18 infection and subsequent dysplasia. Stage I disease, which accounts for almost half of staging at presentation, is defined by carcinoma confined to the cervix and has a >80% 5-year survival (Figure III-44).
+
+
++
+
++
++
++
Which of the following statements regarding the presentation and evaluation of suspected brain malignancy is true?
++
++
++
A. A low-grade glioma is more likely to present with a new seizure than a high-grade glioma.
++
++
B. Approximately half of all malignant brain lesions are metastatic, with the other half being all primary brain tumors combined.
++
++
C. CT with intravenous contrast is the preferred radiologic study to evaluate a suspected intracranial tumor.
++
++
D. Headache is present at presentation in over 75% of patients with brain tumors.
++
++
E. High-grade gliomas are more likely to present with headache and alteration in cognitive function than metastatic lesions.
+
++
The answer is A. (Chap. 118) Primary brain tumors are diagnosed in approximately 52,000 people each year in the United States. At least one-half of these tumors are malignant and associated with a high mortality. Glial tumors account for about 30% of all primary brain tumors, and 80% of those are malignant. Meningiomas account for 35%, vestibular schwannomas 10%, and central nervous system (CNS) lymphomas about 2%. Brain metastases are three times more common than all primary brain tumors combined and are diagnosed in approximately 150,000 people each year. Brain tumors of any type can present with a variety of symptoms and signs that fall into two categories: general and focal; patients often have a combination of the two (Table III-45).
++
++
Seizures are a common presentation of brain tumors, occurring in about 25% of patients with brain metastases or malignant gliomas, but can be the presenting symptom in up to 90% of patients with a low-grade glioma. All seizures that arise from a brain tumor will have a focal onset whether or not it is apparent clinically. Cranial MRI is the preferred diagnostic test for any patient suspected of having a brain tumor and should be performed with gadolinium contrast administration. CT scan should be reserved for patients unable to undergo MRI (e.g., pacemaker). Malignant brain tumors—whether primary or metastatic—typically enhance with gadolinium and may have central areas of necrosis; they are characteristically surrounded by edema of the neighboring white matter. Low-grade gliomas usually do not enhance with gadolinium and are best appreciated on fluid-attenuated inversion recovery (FLAIR) MRIs. Meningiomas have a characteristic appearance on MRI because they are dural based with a dural tail and compress, but do not invade, the brain.
++
++
++
A 63-year-old woman presents to the emergency department after developing a new-onset seizure. Family members came to her when they heard a commotion in her bedroom and found her having a tonic-clonic seizure that spontaneously ended after about 1 minute. She has no prior neurologic history, and her medical history is only notable for hypertension controlled with a diuretic and 40 pack-years of cigarette smoking. There is no history of illicit drug use, and she has been in excellent health until this episode. She works as a congressional staffer in a legislative office. Her physical examination is unremarkable, and she is somnolent and disoriented after receiving lorazepam by the emergency medical team. She receives an urgent head MRI with contrast (Figure III-46). The largest lesion visualized is less than 3 cm in diameter. Which of the following statements is true about her likely diagnosis?
+
+
+
++
+
++
++
++
A. Metastatic lesions due to ovarian carcinoma are more likely to be present than metastatic lesions due to a lung carcinoma.
++
++
B. She is likely a candidate for platinum-based chemotherapy.
++
++
C. She likely has a primary brain malignancy.
++
++
D. She may be a candidate for stereotactic radiosurgery.
++
++
E. Stereotactic radiosurgery and whole-brain radiation have similar mortality outcomes for metastatic disease.
+
++
The answer is D. (Chap. 118) This patient presents with at least two presumed metastatic lesions in the right frontal and right cerebellar lobes. The multiple lesions make a primary brain tumor unlikely. The distribution of metastases in the brain approximates the proportion of blood flow such that about 85% of all metastases are supratentorial and 15% occur in the posterior fossa. The most common sources of brain metastases are lung and breast carcinomas; melanoma has the greatest propensity to metastasize to the brain, being found in 80% of patients at autopsy. The standard treatment for brain metastases has been whole-brain radiotherapy (WBRT) usually administered to a total dose of 3000 cGy in 10 fractions. This affords rapid palliation, and approximately 80% of patients improve with glucocorticoids and radiotherapy. However, it is not curative. Median survival is only 4–6 months. More recently, stereotactic radiosurgery (SRS) delivered through a variety of techniques, including the gamma knife, linear accelerator, proton beam, and CyberKnife, can deliver highly focused doses of radiation therapy, usually in a single fraction. SRS can effectively sterilize the visible lesions and afford local disease control in 80%–90% of patients. In addition, there are some patients who have clearly been cured of their brain metastases using SRS, whereas this is distinctly rare with WBRT. However, SRS can be used only for lesions 3 cm or less in diameter and should be confined to patients with only one to three metastases. The addition of WBRT to SRS improves disease control in the nervous system but does not prolong survival. Randomized controlled trials have demonstrated that surgical extirpation of a single brain metastasis followed by WBRT is superior to WBRT alone. Removal of two lesions or a single symptomatic mass, particularly if compressing the ventricular system, can also be useful. Chemotherapy is rarely useful for brain metastases.
++
++
++
++
A 55-year-old woman presents to the emergency department after a minor motorcycle collision complaining of diffuse chest pain. Her chest radiograph shows multiple 2- to 4-cm nodules and masses without cavitation in all lobes. Her physical examination is totally normal other than some diffuse chest. She has no past medical history and takes no medications, other than a multivitamin. She exercises regularly and had a negative colonoscopy and mammogram within the last 2 years. She works as a librarian and rides motorcycles for recreation. There is no history of cigarette or illicit drug use. Abdominal, pelvic, and head imaging shows no likely primary lesions. A bronchoscopic biopsy of a lung lesion is performed and shows histology consistent with moderately well-differentiated adenocarcinoma. There were no airway abnormalities. Her FDG-PET scan shows no lesions other than those in the lung, and repeat colonoscopy and mammography are normal. All of the following statements regarding her carcinoma are true EXCEPT:
++
++
++
A. Gene expression profiles may help determine the original primary carcinoma and aid in determining the most appropriate therapy.
++
++
B. Immunohistochemical staining of cytokeratin 7 (CK 7) and cytokeratin 20 (CK 20) may help determine the most appropriate therapy.
++
++
C. Median survival of patients with carcinoma of uncertain primary is approximately 18 months.
++
++
D. Moderately differentiated adenocarcinoma is the most common histology of carcinoma of unknown primary.
++
++
E. Prognostic factors including performance status and LDH level may identify patients most amenable to therapy.
+
++
The answer is C. (Chap. 120e) Carcinoma of unknown primary (CUP) is a biopsy-proven malignancy for which the anatomic site of origin remains unidentified after an intensive search. CUP is one of the 10 most frequently diagnosed cancers worldwide, accounting for 3%–5% of all cancers. CUP is limited to epithelial cancers and does not include lymphomas, metastatic melanomas, and metastatic sarcomas because these cancers have specific histology- and stage-based treatments that guide management. The emergence of sophisticated imaging, robust immunohistochemistry (IHC), and genomic and proteomic tools has challenged the “unknown” designation. Additionally, effective targeted therapies in several cancers have moved the paradigm from empiricism to considering an individualized approach to CUP management. The reasons cancers present as CUP remain unclear. One hypothesis is that the primary tumor either regresses after seeding the metastasis or remains so small that it is not detected. It is possible that CUP falls on the continuum of cancer presentation where the primary has been contained or eliminated by the natural immune defenses. Alternatively, CUP may represent a specific malignant event that results in an increase in metastatic spread or survival relative to the primary tumor. Most tumor markers, including CEA, CA-125, CA 19-9, and CA 15-3, when elevated, are nonspecific and not helpful in determining the primary tumor site. Men who present with adenocarcinoma and osteoblastic metastasis should undergo a PSA test. In patients with undifferentiated or poorly differentiated carcinoma (especially with a midline tumor), elevated β-hCG and AFP levels suggest the possibility of an extragonadal germ cell (testicular) tumor. Monoclonal antibodies to specific cytokeratin (CK) subtypes have been used to help classify tumors according to their site of origin; commonly used CK stains in adenocarcinoma CUP are CK7 and CK20. CK7 is found in tumors of the lung, ovary, endometrium, breast, and upper gastrointestinal tract including pancreaticobiliary cancers, whereas CK20 is normally expressed in the gastrointestinal epithelium, urothelium, and Merkel cells. Gene expression profiling offers the promise of substantially increasing the yield of CUP.
++
The median survival duration of most patients with disseminated CUP is approximately 6–10 months. Systemic chemotherapy is the primary treatment modality in most patients with disseminated disease, but the careful integration of surgery, radiation therapy, and even periods of observation is important in the overall management of this condition (Figure III-47). Prognostic factors include performance status, site and number of metastases, response to chemotherapy, and serum LDH levels.
+
+
++
FIGURE III-47 C, chemotherapy; CRT, chemoradiation; CUP, cancer of unknown primary; GI, gastrointestinal; IHC, immunohistochemistry; MRI, magnetic resonance imaging; PSA, prostate-specific antigen; RT, radiation.
+
++
++
++
A 63-year-old woman is brought to the emergency department by her nephew because of severe confusion and obtundation. Her vital signs are normal, and there are no focal physical findings. She is found to have hypercalcemia with a serum level of 14.8 mg/dL, along with minimal elevation of blood urea nitrogen (BUN) and creatinine. Initial evaluation reveals a chest radiograph with multiple nodules suggestive of metastatic disease. Unfortunately the nephew does not know anything about his aunt’s medical history. He reports that she was in town attending a healing yoga conference. Subsequent laboratory testing reveals a normal parathyroid hormone level and an elevated parathyroid hormone–related protein level. All of the following are a likely primary malignancy in this woman EXCEPT:
++
++
++
A. Adenocarcinoma of the breast
++
++
++
++
C. Squamous cell of the lung
++
++
D. Squamous cell of the piriform sinus
++
++
E. Transitional cell of the bladder
+
++
The answer is B. (Chap. 121) Humoral hypercalcemia of malignancy (HHM) occurs in up to 20% of patients with cancer. HHM is most common in cancers of the lung, head and neck, skin, esophagus, breast, and genitourinary tract and in multiple myeloma and lymphomas. There are several distinct humoral causes of HHM, but it is caused most commonly by overproduction of parathyroid hormone–related protein (PTHrP). In addition to acting as a circulating humoral factor, bone metastases (e.g., breast, multiple myeloma) may produce PTHrP, leading to local osteolysis and hypercalcemia. PTHrP is structurally related to parathyroid hormone (PTH) and binds to the PTH receptor, explaining the similar biochemical features of HHM and hyperparathyroidism. Metastatic lesions to bone are more likely to produce PTHrP than are metastases in other tissues. Another relatively common cause of HHM is excess production of 1,25-dihydroxyvitamin D. Like granulomatous disorders associated with hypercalcemia, lymphomas can produce an enzyme that converts 25-hydroxyvitamin D to the more active 1,25-dihydroxyvitamin D, leading to enhanced gastrointestinal calcium absorption. Other causes of HHM include tumor-mediated production of osteolytic cytokines and inflammatory mediators. In this case, the multiple metastatic nodules and elevated PTHrP make lymphoma the least likely malignancy.
++
++
++
A 63-year-old woman is brought to the emergency department by her nephew because of severe confusion and obtundation. Her vital signs are normal, and there are no focal physical findings. She is found to have hypercalcemia with a serum level of 14.8 mg/dL, along with minimal elevation of blood urea nitrogen (BUN) and creatinine. Initial evaluation reveals a chest radiograph with multiple nodules suggestive of metastatic disease. Unfortunately the nephew does not know anything about his aunt’s medical history. He reports that she was in town attending a healing yoga conference. Subsequent laboratory testing reveals a normal parathyroid hormone level and an elevated parathyroid hormone–related protein level. The patient described should receive treatment with all of the following for her hypercalcemia EXCEPT:
++
++
++
++
++
++
++
++
++
++
++
+
++
The answer is E. (Chap. 121) The management of severe HHM begins with saline rehydration (typically 200–500 mL/hr) to dilute serum calcium and promote calciuresis. Forced diuresis with furosemide or other loop diuretics can enhance calcium excretion but provides relatively little value except in life-threatening hypercalcemia. When used, loop diuretics should be administered only after complete rehydration and with careful monitoring of fluid balance. Oral phosphorus should be given until serum phosphorus is >1 mmol/L (>3 mg/dL). Bisphosphonates such as pamidronate, zoledronate, and etidronate can reduce serum calcium within 1–2 days and suppress calcium release for several weeks. Bisphosphonate infusions can be repeated, or oral bisphosphonates can be used for chronic treatment. Dialysis should be considered in severe hypercalcemia when saline hydration and bisphosphonate treatments are not possible or are too slow in onset. Previously used agents such as calcitonin and mithramycin have little utility now that bisphosphonates are available. Calcitonin should be considered when rapid correction of severe hypercalcemia is needed. Hypercalcemia associated with lymphomas, multiple myeloma, or leukemia may respond to glucocorticoid treatment. This patient most likely does not have lymphoma, so initial treatment should not include corticosteroids.
++
++
++
A 61-year-old woman is diagnosed with stage II breast carcinoma. She receives a mastectomy, where she is found to have one positive lymph node. The tumor is positive for estrogen receptor, progestin receptor, and overexpression of HER2/Neu. She receives adjuvant chemotherapy with doxorubicin, cisplatin, and trastuzumab. Match the concerning toxicity with the appropriate agent.
++
++
++
++
A. Reversible cardiomyopathy
++
++
B. Irreversible cardiomyopathy
++
++
C. Sensorimotor neuropathy
+
++
The answer is C. (Chap. 125) Dose-dependent myocardial toxicity of anthracyclines with characteristic myofibrillar dropout is pathologically pathognomonic on endomyocardial biopsy. Anthracycline cardiotoxicity occurs through a root mechanism of chemical free radical damage. Fe3+-doxorubicin complexes damage DNA, nuclear and cytoplasmic membranes, and mitochondria. About 5% of patients receiving >450–550 mg/m2 of doxorubicin will develop congestive heart failure (CHF). Cardiotoxicity in relation to the dose of anthracycline is clearly not a step function, but rather a continuous function, and occasional patients are seen with CHF at substantially lower doses. Advanced age, other concomitant cardiac disease, hypertension, diabetes, and thoracic radiation therapy are all important cofactors in promoting anthracycline-associated CHF. The risk of cardiac failure appears to be substantially lower when doxorubicin is administered by continuous infusion. Anthracycline-related CHF is difficult to reverse and has a mortality rate as high as 50%, making prevention crucial. Monitoring patients for cardiac toxicity typically involves periodic gated nuclear cardiac blood pool ejection fraction testing (multigated acquisition scan [MUGA]) or cardiac ultrasonography. More recently, cardiac MRI has been used, but MRI is not standard or widespread. After anthracyclines, trastuzumab is the next most frequent cardiotoxic drug currently in use. Trastuzumab is frequently used as adjuvant breast cancer therapy, sometimes in conjunction with anthracyclines, which is believed to result in additive or possibly synergistic toxicity. In contrast to anthracyclines, cardiotoxicity is not dose related, is usually reversible, is not associated with pathologic changes of anthracyclines on cardiac myofibrils, and has a different biochemical mechanism inhibiting intrinsic cardiac repair mechanisms. Toxicity is typically routinely monitored every three to four doses using functional cardiac testing as mentioned earlier for anthracyclines. Cisplatin is associated with sensorimotor neuropathy and hearing loss, especially at doses >400 mg/m2, requiring audiometry in patients with preexisting hearing compromise. Carboplatin is often substituted in such cases given its lesser effect on hearing.
++
++
++
A 61-year-old woman is diagnosed with stage II breast carcinoma. She receives a mastectomy, where she is found to have one positive lymph node. The tumor is positive for estrogen receptor, progestin receptor, and overexpression of HER2/Neu. She receives adjuvant chemotherapy with doxorubicin, cisplatin, and trastuzumab. Match the concerning toxicity with the appropriate agent.
++
++
++
++
A. Reversible cardiomyopathy
++
++
B. Irreversible cardiomyopathy
++
++
C. Sensorimotor neuropathy
+
++
The answer is B. (Chap. 125) Dose-dependent myocardial toxicity of anthracyclines with characteristic myofibrillar dropout is pathologically pathognomonic on endomyocardial biopsy. Anthracycline cardiotoxicity occurs through a root mechanism of chemical free radical damage. Fe3+-doxorubicin complexes damage DNA, nuclear and cytoplasmic membranes, and mitochondria. About 5% of patients receiving >450–550 mg/m2 of doxorubicin will develop congestive heart failure (CHF). Cardiotoxicity in relation to the dose of anthracycline is clearly not a step function, but rather a continuous function, and occasional patients are seen with CHF at substantially lower doses. Advanced age, other concomitant cardiac disease, hypertension, diabetes, and thoracic radiation therapy are all important cofactors in promoting anthracycline-associated CHF. The risk of cardiac failure appears to be substantially lower when doxorubicin is administered by continuous infusion. Anthracycline-related CHF is difficult to reverse and has a mortality rate as high as 50%, making prevention crucial. Monitoring patients for cardiac toxicity typically involves periodic gated nuclear cardiac blood pool ejection fraction testing (multigated acquisition scan [MUGA]) or cardiac ultrasonography. More recently, cardiac MRI has been used, but MRI is not standard or widespread. After anthracyclines, trastuzumab is the next most frequent cardiotoxic drug currently in use. Trastuzumab is frequently used as adjuvant breast cancer therapy, sometimes in conjunction with anthracyclines, which is believed to result in additive or possibly synergistic toxicity. In contrast to anthracyclines, cardiotoxicity is not dose related, is usually reversible, is not associated with pathologic changes of anthracyclines on cardiac myofibrils, and has a different biochemical mechanism inhibiting intrinsic cardiac repair mechanisms. Toxicity is typically routinely monitored every three to four doses using functional cardiac testing as mentioned earlier for anthracyclines. Cisplatin is associated with sensorimotor neuropathy and hearing loss, especially at doses >400 mg/m2, requiring audiometry in patients with preexisting hearing compromise. Carboplatin is often substituted in such cases given its lesser effect on hearing.
++
++
++
A 61-year-old woman is diagnosed with stage II breast carcinoma. She receives a mastectomy, where she is found to have one positive lymph node. The tumor is positive for estrogen receptor, progestin receptor, and overexpression of HER2/Neu. She receives adjuvant chemotherapy with doxorubicin, cisplatin, and trastuzumab. Match the concerning toxicity with the appropriate agent.
++
++
++
++
A. Reversible cardiomyopathy
++
++
B. Irreversible cardiomyopathy
++
++
C. Sensorimotor neuropathy
+
++
The answer is A. (Chap. 125) Dose-dependent myocardial toxicity of anthracyclines with characteristic myofibrillar dropout is pathologically pathognomonic on endomyocardial biopsy. Anthracycline cardiotoxicity occurs through a root mechanism of chemical free radical damage. Fe3+-doxorubicin complexes damage DNA, nuclear and cytoplasmic membranes, and mitochondria. About 5% of patients receiving >450–550 mg/m2 of doxorubicin will develop congestive heart failure (CHF). Cardiotoxicity in relation to the dose of anthracycline is clearly not a step function, but rather a continuous function, and occasional patients are seen with CHF at substantially lower doses. Advanced age, other concomitant cardiac disease, hypertension, diabetes, and thoracic radiation therapy are all important cofactors in promoting anthracycline-associated CHF. The risk of cardiac failure appears to be substantially lower when doxorubicin is administered by continuous infusion. Anthracycline-related CHF is difficult to reverse and has a mortality rate as high as 50%, making prevention crucial. Monitoring patients for cardiac toxicity typically involves periodic gated nuclear cardiac blood pool ejection fraction testing (multigated acquisition scan [MUGA]) or cardiac ultrasonography. More recently, cardiac MRI has been used, but MRI is not standard or widespread. After anthracyclines, trastuzumab is the next most frequent cardiotoxic drug currently in use. Trastuzumab is frequently used as adjuvant breast cancer therapy, sometimes in conjunction with anthracyclines, which is believed to result in additive or possibly synergistic toxicity. In contrast to anthracyclines, cardiotoxicity is not dose related, is usually reversible, is not associated with pathologic changes of anthracyclines on cardiac myofibrils, and has a different biochemical mechanism inhibiting intrinsic cardiac repair mechanisms. Toxicity is typically routinely monitored every three to four doses using functional cardiac testing as mentioned earlier for anthracyclines. Cisplatin is associated with sensorimotor neuropathy and hearing loss, especially at doses >400 mg/m2, requiring audiometry in patients with preexisting hearing compromise. Carboplatin is often substituted in such cases given its lesser effect on hearing.
++
++
++
Which of the following proteins is most responsible for iron transport in the plasma?
++
++
++
++
++
++
++
++
++
++
++
+
++
The answer is E. (Chap. 126) Iron absorbed from the diet or released from stores circulates in the plasma bound to transferrin, the iron transport protein. Transferrin that carries iron exists in two forms—monoferric (one iron atom) or diferric (two iron atoms). The turnover (half-clearance time) of transferrin-bound iron is very rapid—typically 60–90 minutes. Because almost all of the iron transported by transferrin is delivered to the erythroid marrow, the clearance time of transferrin-bound iron from the circulation is affected most by the plasma iron level and the erythroid marrow activity. Ferritin is an iron storage protein, and levels represent an indirect measure of total-body iron stores. Free iron, which is toxic to cells, may also be stored intracellularly bound to hemosiderin. Haptoglobin binds free hemoglobin in the plasma, and low levels are consistent intravascular hemolysis. Albumin, which binds many serum proteins, does not bind significant amounts of free iron in plasma.
++
++
++
A 38-year-old woman with a history of inflammatory bowel disease complains of worsening fatigue over the last 1–2 months. Her gastrointestinal disease has been stable for the past year on infliximab. Physical examination is unremarkable, including heme-negative stool. Her hemoglobin has fallen from 11 g/dL to 8 g/dL since last checked 6 months ago. Additionally, her serum iron and ferritin, which were both normal 6 months ago, are now low. Her blood smear is shown in Figure III-52. Which of the following is the most likely etiology of her new anemia?
+
+
++
FIGURE III-52 From Lichtman M, Beutler E, Kaushansky K, Kipps T (eds): Williams Hematology, 7th ed, New York, NY: McGraw-Hill, 2005.
+
++
++
++
++
++
++
++
++
++
++
++
E. Vitamin B12 deficiency
+
++
The answer is C. (Chap. 126) The peripheral blood smear is consistent with a hypochromic microcytic anemia. There are only three conditions that need to be considered in the differential diagnosis of hypochromic microcytic anemia: thalassemia, inflammation, and sideroblastic anemia related to a myelodysplastic syndrome. Thalassemia is an inherited defect of globin chain synthesis and will have normal or increased serum iron levels. The anemia of inflammation (AI; also referred to as the anemia of chronic disease) is characterized by inadequate iron supply to the erythroid marrow. The distinction between true iron deficiency anemia and AI is a common diagnostic problem. Usually, AI is normocytic and normochromic. The iron values usually make the differential diagnosis clear, as the ferritin level is normal or increased and the percent transferrin saturation and total iron-binding capacity are typically below normal. Sideroblastic anemia is the least common microcytic hypochromic anemia condition. Occasionally, patients with myelodysplasia have impaired hemoglobin synthesis with mitochondrial dysfunction, resulting in impaired iron incorporation into heme. The iron values again reveal normal stores and more than an adequate supply to the marrow, despite the microcytosis and hypochromia. Vitamin B12 deficiency and folate deficiency cause macrocytic anemia. This patient has low iron and low ferritin, consistent with iron deficiency.
++
++
++
++
All of the following statements regarding hemoglobinopathies are true EXCEPT:
++
++
++
A. Approximately 15% of African Americans are silent carriers of α-thalassemia.
++
++
B. Approximately 15% of African Americans are heterozygous for sickle cell disease.
++
++
C. Hemoglobinopathies are especially common in areas where malaria is endemic.
++
++
D. Sickle cell disease is the most common structural hemoglobinopathy.
++
++
E. Thalassemias are the most common genetic disorders in the world.
+
++
The answer is B. (Chap. 127) Hemoglobinopathies are especially common in areas in which malaria is endemic. This clustering of hemoglobinopathies is assumed to reflect a selective survival advantage for the abnormal red blood cell (RBC), which presumably provides a less hospitable environment during the obligate RBC stages of the parasitic life cycle. Very young children with α-thalassemia are more susceptible to infection with the nonlethal Plasmodium vivax. Thalassemia might then favor a natural protection against infection with the more lethal Plasmodium falciparum.
++
Thalassemias are the most common genetic disorders in the world, affecting nearly 200 million people worldwide. About 15% of African Americans are silent carriers for α-thalassemia; α-thalassemia trait (minor) occurs in 3% of African American and in 1%–15% of persons of Mediterranean origin. β-Thalassemia has a 10%–15% incidence in individuals from the Mediterranean and Southeast Asia and 0.8% in African Americans. The number of severe cases of thalassemia in the United States is about 1000. Sickle cell disease is the most common structural hemoglobinopathy, occurring in heterozygous form in approximately 8% of African Americans and in homozygous form in 1 in 400. Between 2% and 3% of African Americans carry a hemoglobin C allele.
++
++
++
A 22-year-old man with known sickle cell anemia is admitted to the intensive care unit with diffuse body pain, shortness of breath, fever, and cough. He started having a bone pain crisis 1 day ago and tried to treat it at home with oral hydration. On examination, his blood pressure and heart rate are elevated, and he is in obvious pain and respiratory discomfort. His room air arterial oxygen saturation (SaO2) is 83% and increases to 91% on a nonrebreather oxygen face mask. His chest radiograph shows bilateral diffuse alveolar infiltrates. This is his third similar episode in the last 12 months. All of the statements regarding his condition are true EXCEPT:
++
++
++
A. Chronic therapy with oral hydroxyurea should be considered.
++
++
B. He is having a sickle cell acute chest syndrome.
++
++
C. He should receive daily sildenafil.
++
++
D. Hematocrit should be maintained at >30%.
++
++
E. Hydration should be continued.
+
++
The answer is C. (Chap. 127) Acute chest syndrome is a distinctive manifestation characterized by chest pain, tachypnea, fever, cough, and arterial oxygen desaturation. It can mimic pneumonia, pulmonary emboli, bone marrow infarction and embolism, myocardial ischemia, or in situ lung infarction. Acute chest syndrome is thought to reflect in situ sickling within the lung, producing pain and temporary pulmonary dysfunction. Often it is difficult or impossible to distinguish among other possibilities. Pulmonary infarction and pneumonia are the most frequent underlying or concomitant conditions in patients with this syndrome. Repeated episodes of acute chest pain correlate with reduced survival. Chronic acute or subacute pulmonary crises lead to pulmonary hypertension and cor pulmonale, an increasingly common cause of death as patients survive longer. Considerable controversy exists about the possible role played by free plasma hemoglobin S in scavenging nitrogen dioxide (NO2), thus raising pulmonary vascular tone. Trials of sildenafil to restore NO2 levels were terminated because of adverse effects. Acute chest syndrome is a medical emergency that may require management in an intensive care unit. Hydration should be monitored carefully to avoid the development of pulmonary edema, and oxygen therapy should be administered to avoid hypoxemia. Critical interventions are transfusion to maintain a hematocrit >30% and emergency exchange transfusion if arterial saturation drops to <90%. The most significant advance in the therapy of sickle cell anemia has been the introduction of hydroxyurea as a mainstay of therapy for patients with severe symptoms. Hydroxyurea (10–30 mg/kg/d) increases fetal hemoglobin and may also exert beneficial effects on RBC hydration, vascular wall adherence, and suppression of the granulocyte and reticulocyte counts; dosage is titrated to maintain a white cell count between 5000 and 8000/μL. White cells and reticulocytes may play a major role in the pathogenesis of sickle cell crisis, and their suppression may be an important side benefit of hydroxyurea therapy.
++
Hydroxyurea should be considered in patients experiencing repeated episodes of acute chest syndrome or with more than three crises per year requiring hospitalization. Hydroxyurea offers broad benefits to most patients whose disease is severe enough to impair their functional status, and it may improve survival. Fetal hemoglobin levels increase in most patients within a few months.
++
++
++
A 28-year-old woman is referred to your clinic for evaluation of anemia that was found on a life insurance screening examination. She reports being healthy, takes no medications other than a multivitamin with iron, and only admits some recent fatigue on exertion in the past 6–9 months. She menstruates regularly with 3- to 4-day menses that have not changed in years. She eats a normal omnivorous diet, smokes one pack per day of cigarettes, and does not use illicit drugs. The results of her complete blood count (CBC) reveal a white blood cell (WBC) count of 4.0/μL, platelet count of 235,000/μL, and hemoglobin of 8 g/dL with a mean corpuscular volume (MCV) of 105. Her blood smear is shown in Figure III-55. She has normal renal and liver function. Which of the following is the most likely diagnosis?
+
+
++
FIGURE III-55 Reprinted from Hoffbrand AV, Catovsky D, Tuddenham EGD (eds): Postgraduate Haematology, 5th ed. Oxford, UK: Blackwell Publishing, 2005; with permission.
+
++
++
++
A. Acute myeloblastic anemia
++
++
B. Hereditary spherocytosis
++
++
C. Iron deficiency anemia
++
++
D. Nutritional megaloblastic anemia
++
++
+
++
The answer is E. (Chap. 128) The peripheral blood smear shows macrocytosis, anisocytosis, and poikilocytosis with a hypersegmented (greater than five lobes) polymorphonuclear cell. These findings are diagnostic of megaloblastic anemia. In cases of severe anemia, there may also be reduction in white blood cell (WBC) and platelet count. The most common causes of megaloblastic anemia are folate and cobalamin deficiency (Tables III-55 and III-56). This patient most likely has pernicious anemia given her normal diet and absence of other symptoms. The next best test will be a serum vitamin B12 level. In patients with cobalamin deficiency sufficient to cause anemia or neuropathy, the serum methylmalonic acid (MMA) level is raised. Sensitive methods for measuring MMA and homocysteine in serum have been introduced and recommended for the early diagnosis of cobalamin deficiency, even in the absence of hematologic abnormalities or subnormal levels of serum cobalamin. Serum MMA levels fluctuate, however, in patients with renal failure. Serum homocysteine is raised in both early cobalamin and folate deficiency but may be raised in other conditions (e.g., chronic renal disease, alcoholism, smoking, pyridoxine deficiency, hypothyroidism, and therapy with steroids, cyclosporine, and other drugs).
++
++
++
++
++
A 28-year-old woman is referred to your clinic for evaluation of anemia that was found on a life insurance screening examination. She reports being healthy, takes no medications other than a multivitamin with iron, and only admits some recent fatigue on exertion in the past 6–9 months. She menstruates regularly with 3- to 4-day menses that have not changed in years. She eats a normal omnivorous diet, smokes one pack per day of cigarettes, and does not use illicit drugs. The results of her complete blood count (CBC) reveal a white blood cell (WBC) count of 4.0/μL, platelet count of 235,000/μL, and hemoglobin of 8 g/dL with a mean corpuscular volume (MCV) of 105. Her blood smear is shown in Figure III-55. She has normal renal and liver function. In the patient, which of the following is the next best study to confirm the diagnosis?
+
+
++
FIGURE III-55 Reprinted from Hoffbrand AV, Catovsky D, Tuddenham EGD (eds): Postgraduate Haematology, 5th ed. Oxford, UK: Blackwell Publishing, 2005; with permission.
+
++
++
++
++
++
B. Hemoglobin electrophoresis
++
++
C. Serum homocysteine level
++
++
D. Serum iron and transferrin levels
++
++
E. Serum vitamin B12 level
+
++
The answer is E. (Chap. 128) The peripheral blood smear shows macrocytosis, anisocytosis, and poikilocytosis with a hypersegmented (greater than five lobes) polymorphonuclear cell. These findings are diagnostic of megaloblastic anemia. In cases of severe anemia, there may also be reduction in white blood cell (WBC) and platelet count. The most common causes of megaloblastic anemia are folate and cobalamin deficiency (Tables III-55 and III-56). This patient most likely has pernicious anemia given her normal diet and absence of other symptoms. The next best test will be a serum vitamin B12 level. In patients with cobalamin deficiency sufficient to cause anemia or neuropathy, the serum methylmalonic acid (MMA) level is raised. Sensitive methods for measuring MMA and homocysteine in serum have been introduced and recommended for the early diagnosis of cobalamin deficiency, even in the absence of hematologic abnormalities or subnormal levels of serum cobalamin. Serum MMA levels fluctuate, however, in patients with renal failure. Serum homocysteine is raised in both early cobalamin and folate deficiency but may be raised in other conditions (e.g., chronic renal disease, alcoholism, smoking, pyridoxine deficiency, hypothyroidism, and therapy with steroids, cyclosporine, and other drugs).
++
++
++
++
++
All of the following are typically increased in a patient with hemolytic anemia EXCEPT:
++
++
++
A. Aspartate aminotransferase (AST)
++
++
++
++
++
++
++
++
+
++
The answer is B. (Chap. 129) What differentiates hemolytic anemias (HAs) from other anemias is that the patient has signs and symptoms arising directly from hemolysis. At the clinical level, the main sign is jaundice; in addition, the patient may report discoloration of the urine. In many cases of HA, the spleen is enlarged, because it is a preferential site of hemolysis; and in some cases, the liver may be enlarged as well. The laboratory features of HA are related to hemolysis per se and the erythropoietic response of the bone marrow. Hemolysis regularly produces an increase in unconjugated bilirubin and aspartate aminotransferase (AST) in the serum; urobilinogen will be increased in both urine and stool. If hemolysis is mainly intravascular, the telltale sign is hemoglobinuria (often associated with hemosiderinuria); in the serum, there is hemoglobin, LDH is increased, and haptoglobin is reduced. In contrast, the bilirubin level may be normal or only mildly elevated. The main sign of the erythropoietic response by the bone marrow is an increase in reticulocytes (a test all too often neglected in the initial workup of a patient with anemia). Usually the increase will be reflected in both the percentage of reticulocytes (the more commonly quoted figure) and the absolute reticulocyte count (the more definitive parameter). The increased number of reticulocytes is associated with an increased mean corpuscular volume (MCV) in the blood count. On the blood smear, this is reflected in the presence of macrocytes; there is also polychromasia, and sometimes, one sees nucleated red cells.
++
++
++
A 24-year-old man returns to the acute care clinic complaining of 1 day of worsening malaise, weakness, abdominal pain, and discolored dark urine. He was at the clinic yesterday where he was diagnosed with a possible staphylococcal skin carbuncle and treated with empiric trimethoprim/sulfamethoxazole. His physical examination is notable for mild jaundice, normal vital signs, and no focal findings other than the carbuncle in his left axilla. His CBC shows that his hemoglobin has fallen from 13 g/dL to 8 g/dL and his bilirubin has gone from normal to 3.0 mg/dL. His urine dipstick is positive for bilirubin. A peripheral blood smear is shown in Figure III-58. Which of the following is the most likely cause of his new anemia?
+
+
++
FIGURE III-58 From MA Lichtman et al: Lichtman’s Atlas of Hematology. http://www.accessmedicine.com. Copyright © The McGraw-Hill Companies, Inc. All rights reserved.
+
++
++
++
A. Glucose 6-phosphate dehydrogenase (G6PD) deficiency
++
++
B. Hemolytic-uremic syndrome
++
++
C. Hereditary spherocytosis
++
++
D. Iron deficiency anemia
++
++
E. Thrombotic thrombocytopenic purpura (TTP)
+
++
The answer is A. (Chap. 129) The peripheral blood smear shows bite cells (arrow in Figure III-58), anisocytosis, and spherocytes. The combination of the smear, jaundice, and hyperbilirubinemia 1 day after receiving sulfamethoxazole makes glucose 6-phosphate dehydrogenase (G6PD) deficiency most likely. G6PD deficiency is widely distributed in tropical and subtropical parts of the world (Africa, southern Europe, the Middle East, Southeast Asia, and Oceania) and wherever people from those areas have migrated. A conservative estimate is that at least 400 million people have a G6PD deficiency gene. In several of these areas, the frequency of a G6PD deficiency gene may be as high as 20% or more. The G6PD gene is X-linked; thus, males have only one G6PD gene (i.e., they are hemizygous for this gene); therefore, they must be either normal or G6PD deficient. Hemolytic anemia in patients with G6PD deficiency can develop as a result of fava beans, infections, and drugs. Common drugs include primaquine, dapsone, sulfamethoxazole, and nitrofurantoin. Typically, a hemolytic attack starts with malaise, weakness, and abdominal or lumbar pain. After an interval of several hours to 2–3 days, the patient develops jaundice and often dark urine. The onset can be extremely abrupt, especially with favism in children. There may be hemoglobinemia, hemoglobinuria, high LDH, and low or absent plasma haptoglobin. G6PD deficiency may be diagnosed by semiquantitative or quantitative RBC tests or by DNA testing. Hemolytic-uremic syndrome and thrombotic thrombocytopenic purpura cause microangiopathic hemolytic anemia with prominent schistocytes. Iron deficiency causes a microcytic and hypochromic anemia.
++
++
++
You are seeing Ms. Stoked, a 19-year-old collegiate rower who has been in excellent health and takes no chronic medications, for new-onset severe fatigue and left upper abdominal pain. She was treated 2 days ago in the health center for presumed gonorrhea with ceftriaxone and azithromycin. Her past medical history is only notable for one prior episode of presumed gonorrhea when she was 17 years old. Her physical examination is notable for a heart rate of 100 bpm and a palpable spleen. Stool is negative for heme. Laboratory examination is notable for a hemoglobin of 5 g/dL with normal WBC and platelets. Peripheral blood smear shows an excess of spherocytes. Which of the following tests will most likely confirm the diagnosis?
++
++
++
A. ADAMTS-13 activity assay
++
++
B. Direct antiglobulin (Coombs) test
++
++
++
++
++
++
E. Hemoglobin electrophoresis
+
++
The answer is B. (Chap. 129) This patient has autoimmune hemolytic anemia (AIHA) with development of warm immunoglobulin G (IgG) antibodies due to ceftriaxone exposure (Table III-59).
++
++
AIHA is a serious condition; without appropriate treatment, it may have a mortality of approximately 10%. The onset is often abrupt and can be dramatic. The hemoglobin level can drop within days to as low as 4 g/dL; the massive red cell removal will produce jaundice; and sometimes the spleen is enlarged. When this triad is present, the suspicion of AIHA must be high. The diagnostic test for AIHA is the direct antiglobulin (Coombs) test, which detects the presence of antibody on the red cells. When the test is positive, it clinches the diagnosis; when it is negative, the diagnosis is unlikely. The immediate treatment almost invariably includes transfusion of red cells. This may pose a special problem because, if the antibody involved is nonspecific, all of the blood units cross-matched will be incompatible. Whenever the anemia is not immediately life threatening, blood transfusion should be withheld (because compatibility problems may increase with each unit of blood transfused), and medical treatment started immediately with prednisone (1 mg/kg/d), which will produce a remission promptly in at least one-half of patients. Rituximab (anti-CD20) was regarded as second-line treatment, but it is increasingly likely that a relatively low dose (100 mg/wk × 4) of rituximab together with prednisone will become a first-line standard. It is especially encouraging that this approach seems to reduce the rate of relapse, a common occurrence in AIHA. G6PD deficiency is unlikely in this previously healthy female only exposed to ceftriaxone. ADAMTS-13 activity assay is used to diagnose thrombotic thrombocytopenic purpura. Flow cytometry may be used to diagnose paroxysmal nocturnal hemoglobinuria. Hemoglobin electrophoresis is used to diagnose congenital hemoglobinopathies.
++
++
++
A 73-year-old man comes to primary care clinic complaining of 4–6 weeks of increasing malaise, fatigue, dyspnea on exertion, and occasional night sweats. His past medical history is notable for hypertension and hyperlipidemia. Medications include lisinopril and atorvastatin. He had a screening colonoscopy 6 months ago, at which time his laboratory studies were normal. His physical examination today is only notable for a heart rate of 105 bpm and pale mucous membranes. Lungs are clear, and there are no new cardiac findings. Electrocardiogram (ECG) shows sinus tachycardia but no acute changes. Laboratory studies show normal electrolytes, but his WBC is 1300/μL, platelet count is 35,000/μL, and hemoglobin is 7.5 g/dL. Examination of his peripheral smear confirms the pancytopenia and notes the red blood cells are macrocytic, and there are 3% blasts present. All of the following statements regarding his condition are true EXCEPT:
++
++
++
A. Azacitidine may improve his blood counts and prolong survival.
++
++
B. Children with Down syndrome are also at risk.
++
++
C. He is at high risk of developing acute myeloid leukemia.
++
++
D. Hematopoietic stem cell transplantation is contraindicated.
++
++
E. Increasing percentages of bone marrow blasts correlate with worsening prognosis.
+
++
The answer is D. (Chap. 130) This patient likely has idiopathic myelodysplastic syndrome. The myelodysplastic syndromes (MDS) are a heterogeneous group of hematologic disorders broadly characterized by both (1) cytopenias due to bone marrow failure and (2) a high risk of development of acute myeloid leukemia (AML; Table III-60). Anemia, often with thrombocytopenia and neutropenia, occurs with dysmorphic (abnormal-appearing) and usually cellular bone marrow, which is evidence of ineffective blood cell production. In patients with “low-risk” MDS, marrow failure dominates the clinical course. In other patients, myeloblasts are present at diagnosis, chromosomes are abnormal, and the “high risk” is due to leukemic progression. MDS may be fatal due to the complications of pancytopenia or the incurability of leukemia, but a large proportion of patients will die of concurrent disease, the comorbidities typical in an elderly population. Idiopathic MDS is a disease of the elderly; the mean age at onset is older than 70 years. There is a slight male predominance. MDS is a relatively common form of bone marrow failure, with reported incidence rates of 35 to >100 per million persons in the general population and 120 to >500 per million persons in older adults. MDS is rare in children but is increased in children with Down syndrome. MDS is associated with environmental exposures such as radiation and benzene. Secondary MDS occurs as a late toxicity of cancer treatment, usually a combination of radiation and the radiomimetic alkylating agents such as busulfan, nitrosourea, or procarbazine (with a latent period of 5–7 years) or the DNA topoisomerase inhibitors (2-year latency). Anemia dominates the early course of MDS. Most symptomatic patients complain of the gradual onset of fatigue and weakness, dyspnea, and pallor, but at least one-half of patients are asymptomatic, and their MDS is discovered only incidentally on routine blood counts. The physical examination is typical for signs of anemia, with approximately 20% of patients having splenomegaly. Some unusual skin lesions, including Sweet syndrome (febrile neutrophilic dermatosis), occur with MDS. Median survival times of patients with MDS vary widely, but prognosis worsens with increasing percent blasts in bone marrow, cytogenetic abnormalities, and lineages affected by cytopenia. Only hematopoietic stem cell transplantation offers cure of MDS. The current survival rate in selected patient cohorts is approximately 50% at 3 years and is improving. New epigenetic modulator drugs are believed to act through a demethylating mechanism to alter gene regulation and allow differentiation to mature blood cells from the abnormal MDS stem cell. Azacitidine and decitabine are two epigenetic modifiers frequently used in bone marrow failure clinics. Azacitidine improves blood counts and survival in MDS, compared to best supportive care.
++
++
++
++
All of the following disorders are considered by World Health Organization classification as chronic myeloproliferative neoplasms EXCEPT:
++
++
++
A. Chronic myeloid leukemia (Bcr-Abl positive)
++
++
B. Essential thrombocytosis
++
++
++
++
++
++
E. All are chronic myeloproliferative neoplasms
+
++
The answer is E. (Chap. 131) The World Health Organization (WHO) classification of the chronic myeloproliferative neoplasms (MPNs) includes eight disorders, some of which are rare or poorly characterized but all of which share an origin in a multipotent hematopoietic progenitor cell; overproduction of one or more of the formed elements of the blood without significant dysplasia; and a predilection to extramedullary hematopoiesis, myelofibrosis, and transformation at varying rates to acute leukemia (Table III-61).
++
++
Within this broad classification, however, significant phenotypic heterogeneity exists. Some diseases such as chronic myelogenous leukemia (CML), chronic neutrophilic leukemia (CNL), and chronic eosinophilic leukemia (CEL) express primarily a myeloid phenotype, whereas in other diseases, such as polycythemia vera (PV), primary myelofibrosis (PMF), and essential thrombocytosis (ET), erythroid or megakaryocytic hyperplasia predominates. The latter three disorders, in contrast to the former three, also appear capable of transforming into each other. Such phenotypic heterogeneity has a genetic basis; CML is the consequence of the balanced translocation between chromosomes 9 and 22 [t(9;22)(q34;11)]; CNL has been associated with a t(15;19) translocation; and CEL occurs with a deletion or balanced translocations involving the PDGFRα gene. By contrast, to a greater or lesser extent, PV, PMF, and ET are characterized by a mutation, V617F, that causes constitutive activation of JAK2, a tyrosine kinase essential for the function of the erythropoietin and thrombopoietin receptors but not the granulocyte colony-stimulating factor receptor. This important distinction is also reflected in the natural histories of CML, CNL, and CEL, which are usually measured in years, and their high rate of leukemic transformation. By contrast, the natural history of PV, PMF, and ET is usually measured in decades, and transformation to acute leukemia is uncommon in PV and ET in the absence of exposure to mutagenic drugs.
++
++
++
A 53-year-old man is sent to you for evaluation of an elevated hematocrit found incidentally on lab testing. He was removing dry wall in a house and cut his upper arm. A CBC before his arm was stitched revealed a hematocrit of 59% with hemoglobin of 20 g/dL, WBC count of 15.4/μL with a normal differential, and platelet count of 445,000/μL. His physical examination is notable for a room air oxygen saturation of 95%, blood pressure of 145/85 mmHg, and a palpable spleen. He has no past medical history, is a nonsmoker, drinks rarely on social occasions, and is taking no medications. The patient reports his last contact with a physician was 2–3 years ago, and he recalls no abnormalities reported on blood testing. Which of the following is the next diagnostic step?
++
++
++
++
++
B. Erythropoietin (EPO) level
++
++
C. Red blood cell (RBC) mass
++
++
D. Pulmonary function tests
++
++
+
++
The answer is C. (Chaps. 77 and 131) This patient has polycythemia, likely due to PV, with the elevation of hemoglobin, WBC count, and platelets. The next step in his evaluation is measurement of RBC mass. After finding an elevated RBC mass, measurement of erythropoietin levels will distinguish between PV and other causes of polycythemia such as renal cell carcinoma, lung disease, hypoxemic states, or chronic carbon monoxide poisoning (Figure III-62).
+
+
++
FIGURE III-62 AV, atrioventricular; COPD, chronic obstructive pulmonary disease; CT, computed tomography; EPO, erythropoietin; hct, hematocrit; hgb, hemoglobin; IVP, intravenous pyelogram; RBC, red blood cell.
+
++
PV is a clonal disorder involving a multipotent hematopoietic progenitor cell in which phenotypically normal red cells, granulocytes, and platelets accumulate in the absence of a recognizable physiologic stimulus. The etiology of PV is unknown. A mutation in the autoinhibitory pseudokinase domain of the tyrosine kinase JAK2, that replaces valine with phenylalanine (V617F) causing constitutive kinase activation, appears to have a central role in the pathogenesis of PV.
++
++
++
A 53-year-old man is sent to you for evaluation of an elevated hematocrit found incidentally on lab testing. He was removing dry wall in a house and cut his upper arm. A CBC before his arm was stitched revealed a hematocrit of 59% with hemoglobin of 20 g/dL, WBC count of 15.4/μL with a normal differential, and platelet count of 445,000/μL. His physical examination is notable for a room air oxygen saturation of 95%, blood pressure of 145/85 mmHg, and a palpable spleen. He has no past medical history, is a nonsmoker, drinks rarely on social occasions, and is taking no medications. The patient reports his last contact with a physician was 2–3 years ago, and he recalls no abnormalities reported on blood testing. In the patient described, if secondary causes of elevated hemoglobin are ruled out, which of the following is the recommended therapy?
++
++
++
++
++
++
++
++
++
D. Phlebotomy to maintain hemoglobin <14 g/dL
++
++
+
++
The answer is D. (Chap. 131) The patient described had a red cell mass that was markedly elevated, with a low erythropoietin level confirming the diagnosis of PV. PV is generally an indolent disorder, the clinical course of which is measured in decades, and its management should reflect its tempo. Thrombosis due to erythrocytosis is the most significant complication and often the presenting manifestation, and maintenance of the hemoglobin level at ≤140 g/L (14 g/dL; hematocrit <45%) in men and ≤120 g/L (12 g/dL; hematocrit <42%) in women is mandatory to avoid thrombotic complications. Phlebotomy serves initially to reduce hyperviscosity by bringing the red cell mass into the normal range while further expanding the plasma volume. Periodic phlebotomies thereafter serve to maintain the red cell mass within the normal range and to induce a state of iron deficiency that prevents an accelerated reexpansion of the red cell mass. Neither phlebotomy nor iron deficiency increases the platelet count relative to the effect of the disease itself, and thrombocytosis is not correlated with thrombosis in PV, in contrast to the strong correlation between erythrocytosis and thrombosis in this disease. The use of salicylates as a tonic against thrombosis in PV patients is not only potentially harmful if the red cell mass is not controlled by phlebotomy, but is also an unproven remedy. Anticoagulants are only indicated when a thrombosis has occurred and can be difficult to monitor if the red cell mass is substantially elevated owing to the artifactual imbalance between the test tube anticoagulant and plasma that occurs when blood from these patients is assayed for prothrombin or partial thromboplastin activity. Imatinib is a Bcr-Abl tyrosine kinase inhibitor typically used in cases of CML and is not efficacious in PV.
++
++
++
Exposure to all of the following has been associated with the development of acute myelogenous leukemia EXCEPT:
++
++
++
++
++
++
++
++
++
D. Herpes virus infection
++
++
+
++
The answer is D. (Chap. 132) Anticancer drugs are the leading cause of therapy-associated acute myelogenous leukemia (AML). Alkylating agent–associated leukemias, such as cyclophosphamide, occur on average 4–6 years after exposure, and affected individuals have aberrations in chromosomes 5 and 7. Topoisomerase II inhibitor–associated leukemias, such as doxorubicin, occur 1–3 years after exposure, and affected individuals often have aberrations involving chromosome 11q23. High-dose radiation, like that experienced by survivors of the atomic bombs in Japan or nuclear reactor accidents, increases the risk of myeloid leukemias that peaks 5–7 years after exposure. Therapeutic radiation alone seems to add little risk of AML but can increase the risk in people also exposed to alkylating agents. Exposure to benzene, a solvent used in the chemical, plastic, rubber, and pharmaceutical industries, is associated with an increased incidence of AML. Smoking and exposure to petroleum products, paint, embalming fluids, ethylene oxide, herbicides, and pesticides have also been associated with an increased risk of AML. In contrast to B-cell lymphomas, there is no direct evidence linking viral infection to AML.
++
++
++
A 64-year-old man presents with 3 weeks of increasing fatigue and bleeding while brushing his teeth. Physical examination is notable for low-grade fever and normal heart rate, blood pressure, and respiratory rate. He has splenomegaly. A CBC shows marked pancytopenia with blasts present on peripheral smear. A bone marrow aspirate and biopsy result in the diagnosis of acute promyelocytic leukemia with the t(15;17)(q22;q12) cytogenetic rearrangement. Which of the following medications that is specific to acute promyelocytic leukemia will be included in his induction chemotherapy?
++
++
++
++
++
++
++
++
++
++
++
+
++
The answer is E. (Chap. 132) Acute promyelocytic leukemia (APL) is a highly curable subtype of acute myelogenous leukemia, and approximately 85% of these patients achieve long-term survival with current approaches. APL has long been shown to be responsive to cytarabine and daunorubicin, but previously, patients treated with these drugs alone frequently died from disseminated intravascular coagulation (DIC) induced by the release of granule components by the chemotherapy-treated leukemia cells. However, the prognosis of APL patients has changed dramatically from adverse to favorable with the introduction of tretinoin, an oral drug that induces the differentiation of leukemic cells bearing the characteristic translocation [t(15;17)], where disruption of the RARA gene encoding a retinoid acid receptor occurs. Tretinoin decreases the frequency of DIC but produces another complication called the APL differentiation syndrome. Occurring within the first 3 weeks of treatment, it is characterized by fever, fluid retention, dyspnea, chest pain, pulmonary infiltrates, pleural and pericardial effusions, and hypoxemia. The syndrome is related to adhesion of differentiated neoplastic cells to the pulmonary vasculature endothelium. Glucocorticoids, chemotherapy, and/or supportive measures can be effective for management of the APL differentiation syndrome. Temporary discontinuation of tretinoin is necessary in cases of severe APL differentiation syndrome (i.e., patients developing renal failure or requiring admission to the intensive care unit due to respiratory distress). The mortality rate of this syndrome is about 10%. Acyclovir is used to treat herpes virus infection. Daunorubicin is an anthracycline chemotherapy agent commonly used in the therapy of AML and ALL; it is not specific to APL. Rituximab is a monoclonal antibody against CD20 used in a wide assortment of autoimmune and malignant diseases. Sildenafil is a PDE5 inhibitor used for treatment of erectile dysfunction and pulmonary arterial hypertension.
++
++
++
All of the following statements are true regarding chronic myelogenous leukemia (CML) EXCEPT:
++
++
++
A. Allogeneic stem cell transplantation is first-line therapy.
++
++
B. Current therapy includes use of a tyrosine kinase inhibitor.
++
++
C. Half of afflicted individuals are children.
++
++
D. The disease is driven by a mutation in the HFE gene.
++
++
E. With treatment, the 5-year median survival is 50%.
+
++
The answer is B. (Chap. 133) Chronic myeloid leukemia (CML) is a clonal hematopoietic stem cell disorder. The disease is driven by the BCR-ABL1 chimeric gene product, a constitutively active tyrosine kinase, resulting from a reciprocal balanced translocation between the long arms of chromosomes 9 and 22, t(9;22) (q34;q11.2), cytogenetically detected as the Philadelphia chromosome. Untreated, the course of CML may be biphasic or triphasic, with an early indolent or chronic phase, followed often by an accelerated phase and a terminal blastic phase. Before the era of selective BCR-ABL1 tyrosine kinase inhibitors (TKIs), the median survival in CML was 3–7 years, and the 10-year survival rate was 30% or less. Introduced into CML therapy in 2000, TKIs, such as imatinib, nilotinib, and dasatinib, have revolutionized the treatment, natural history, and prognosis of CML. Today, the estimated 10-year survival rate with imatinib mesylate, the first BCR-ABL1 TKI approved, is 85%. Allogeneic stem cell transplantation, a curative but risky treatment approach, is now offered as second- or third-line therapy after failure of TKIs. The median age at diagnosis is 55–65 years. CML is uncommon in children; only 3% of patients with CML are younger than 20 years. Mutations of the HFE gene are associated with primary hemochromatosis.
++
++
++
Which of the following is the most common lymphoid malignancy?
++
++
++
A. Acute lymphoid leukemia
++
++
B. Chronic lymphoid leukemia
++
++
++
++
++
++
+
++
The answer is E. (Chap. 134) Non-Hodgkin lymphoma is the most common lymphoid malignancy (Figure III-67).
+
+
++
FIGURE III-67 ALL, acute lymphoid leukemia; CLL, chronic lymphoid leukemia; MALT, mucosa-associated lymphoid tissue.
+
++
Chronic lymphoid leukemia (CLL) is the most prevalent form of leukemia in Western countries. It occurs most frequently in older adults and is exceedingly rare in children. In contrast to CLL, acute lymphoid leukemias (ALLs) are predominantly cancers of children and young adults. The Burkitt leukemia occurring in children in developing countries seems to be associated with infection by the Epstein-Barr virus (EBV) in infancy. The etiology of ALL in adults is also uncertain. ALL is unusual in middle-aged adults but increases in incidence in the elderly. However, acute myelogenous leukemia (AML) is still much more common in older patients. The preponderance of evidence suggests that Hodgkin lymphoma is of B-cell origin. The incidence of Hodgkin lymphoma appears fairly stable, with 9190 new cases diagnosed in 2014 in the United States. Hodgkin lymphoma is more common in whites than in blacks and more common in males than in females. A bimodal distribution of age at diagnosis has been observed, with one peak incidence occurring in patients in their 20s and the other in those in their 80s. Patients in the younger age groups diagnosed in the United States largely have the nodular sclerosing subtype of Hodgkin lymphoma. Elderly patients, patients infected with HIV, and patients in third-world countries more commonly have mixed-cellularity Hodgkin lymphoma or lymphocyte-depleted Hodgkin lymphoma. Non-Hodgkin lymphomas are more frequent in the elderly and more frequent in men. Patients with both primary and secondary immunodeficiency states are predisposed to developing non-Hodgkin lymphomas. These include patients with HIV infection; patients who have undergone organ transplantation; and patients with inherited immune deficiencies, the sicca syndrome, and rheumatoid arthritis. The incidence of non-Hodgkin lymphomas and the patterns of expression of the various subtypes differ geographically. T-cell lymphomas are more common in Asia than in Western countries, whereas certain subtypes of B-cell lymphomas, such as follicular lymphoma, are more common in Western countries. A specific subtype of non-Hodgkin lymphoma, known as the angiocentric nasal T/natural killer (NK) cell lymphoma, has a striking geographic occurrence, being most frequent in southern Asia and parts of Latin America. A number of environmental factors have been implicated in the occurrence of non-Hodgkin lymphoma, including infectious agents, chemical exposures, and medical treatments. Several studies have demonstrated an association between exposure to agricultural chemicals and an increased incidence of non-Hodgkin lymphoma. Patients treated for Hodgkin lymphoma can develop non-Hodgkin lymphoma; it is unclear whether this is a consequence of the Hodgkin lymphoma or its treatment.
++
++
++
All of the following infectious agents are associated with the development of a lymphoid malignancy EXCEPT:
++
++
++
++
++
++
++
++
++
++
++
+
++
The answer is E. (Chap. 134) A number of infectious agents are associated with the development of lymphoid malignancies. JC virus is associated with progressive multifocal leukoencephalopathy (PML) in immunodeficient individuals (Table III-68).
++
++
++
++
A 'dry' aspiration of the bone marrow is consistent with all of the following EXCEPT:
++
++
++
A. Chronic myeloid leukemia
++
++
++
++
C. Metastatic carcinoma infiltration
++
++
++
++
+
++
The answer is E. (Chap. 135e) A “dry tap” is defined as the inability to aspirate bone marrow and is reported in approximately 4% of attempts (Table III-69). It is rare in the case of normal bone marrow. The differential diagnosis includes metastatic carcinoma infiltration (17%), chronic myeloid leukemia (15%), myelofibrosis (14%), hairy cell leukemia (10%), acute leukemia (10%), and lymphomas including Hodgkin disease (9%).
++
++
++
++
All of the following statements are true regarding the criteria to diagnose hypereosinophilic syndrome EXCEPT:
++
++
++
A. Increased bone marrow eosinophils must be demonstrated.
++
++
B. It is not necessary to have increased circulating eosinophils.
++
++
C. Primary myeloid leukemia must be excluded.
++
++
D. Reactive eosinophilia (e.g., parasitic infection, allergy, collagen vascular disease) must be excluded.
++
++
E. There must be <20% myeloblasts in blood or bone marrow.
+
++
The answer is B. (Chap. 135e) The diagnostic criteria for chronic eosinophilic leukemia and the hypereosinophilic syndrome first require the presence of persistent eosinophilia ≥1500/μL in blood, increased marrow eosinophils, and <20% myeloblasts in blood or marrow. Additional disorders that must be excluded include all causes of reactive eosinophilia, primary neoplasms associated with eosinophilia (e.g., T-cell lymphoma, Hodgkin disease, ALL, mastocytosis, CML, AML, myelodysplasia, myeloproliferative syndromes), and T-cell reaction with increased IL-5 or cytokine production. If these entities have been excluded and the myeloid cells show a clonal chromosome abnormality and blast cells (>2%) are present in peripheral blood or are increased in marrow (but <20%), then the diagnosis is chronic eosinophilic leukemia. Patients with hypereosinophilic syndrome and chronic eosinophilic leukemia may be asymptomatic (discovered on routine testing) or present with systemic findings such as fever, shortness of breath, new neurologic findings, or rheumatologic findings. The heart, lungs, and CNS are most often affected by eosinophil-mediated tissue damage.
++
++
++
All of the following statements regarding mastocytosis are true EXCEPT:
++
++
++
A. Elevated serum tryptase suggests aggressive disease.
++
++
B. Eosinophilia is common.
++
++
C. It is often associated with myeloid neoplasm.
++
++
D. Over 90% of cases are confined to the skin.
++
++
E. Urticaria pigmentosa is the most common clinical manifestation.
+
++
The answer is D. (Chap. 135e) Mastocytosis is a proliferation and accumulation of mast cells in one or more organ systems. Only the skin is involved in approximately 80% of cases, with the other 20% being defined as systemic mastocytosis due to the involvement of another organ system. The most common manifestation of mastocytosis is cutaneous urticaria pigmentosa, a maculopapular pigmented rash involving the papillary dermis. Other cutaneous forms include diffuse cutaneous mastocytosis (almost entirely in children) and mastocytoma. Clinical manifestations of systemic mastocytosis are related to either cellular infiltration of organs and/or release of histamine, proteases, eicosanoids, or heparin from mast cells. Therefore, signs and symptoms may include constitutional symptoms, skin manifestations (pruritus, dermatographia, rash), mediator-related symptoms (abdominal pain, flushing, syncope, hypertension, diarrhea), or bone-related symptoms (fracture, pain, arthralgia). In a recent series, 40% of patients with systemic mastocytosis had an associated myeloid neoplasm, most commonly myeloproliferative syndrome, CML, and MDS. Eosinophilia was present in approximately one-third of patients. Elevated serum tryptase, bone marrow involvement, splenomegaly, skeletal involvement, cytopenia, and malabsorption predict more aggressive disease and worse prognosis. Many patients with systemic mastocytosis have an activating mutation of c-KIT, a kinase inhibited by imatinib; however, the mutation appears relatively resistant to this agent.
++
++
++
A 58-year-old man is evaluated for sudden-onset cough with yellow sputum production and dyspnea in the emergency department. Aside from systemic hypertension, he is otherwise healthy. His only medication is amlodipine. Chest radiograph shows a right upper lobe alveolar infiltrate, and labs are notable for a BUN of 53 mg/dL, creatinine of 2.8 mg/dL, calcium of 12.3 mg/dL, total protein of 9 g/dL and albumin of 3.1 g/dL. Sputum culture grows Streptococcus pneumoniae. Which of the following tests will confirm the underlying condition predisposing him to pneumococcal pneumonia?
++
++
++
++
++
B. CT of chest, abdomen, and pelvis with contrast
++
++
++
++
D. Sweat chloride testing
++
++
E. Videoscopic swallow study
+
++
The answer is A. (Chap. 136) The patient presents with pneumococcal pneumonia and evidence of hypercalcemia, renal failure, and a wide protein gap suggestive of an M protein. These findings are classic for multiple myeloma (Table III-72). Although patients appear to be making large quantities of immunoglobulins, they are in fact generally monoclonal, and patients actually have functional hypogammaglobulinemia related to both decreased production and increased destruction of normal antibodies. This hypogammaglobulinemia predisposes patients to infections, most commonly pneumonia with pneumococcus or S aureus or gram-negative pyelonephritis. Bone marrow biopsy would confirm the presence of clonal plasma cells and define the quantity, which will help define treatment options. A serum protein electrophoresis would also be indicated to prove the presence of the M protein suspected by the wide protein gap. While HIV may be associated with kidney injury, both acute and chronic, hypercalcemia would be an unusual feature. There is no clinical history of aspiration, and the location of infiltrate (upper lobe) is unusual for aspiration. Sweat chloride testing is not indicated because there is no suspicion for cystic fibrosis. Because solid organ malignancy is not suspected, CT of the body is unlikely to be helpful.
++
++
++
++
In patients with multiple myeloma, which of the following results is the most powerful predictor of survival?
++
++
++
A. Bone involvement on radiograph
++
++
++
++
++
++
D. Serum β2-microglobulin
++
++
+
++
The answer is D. (Chap. 136) Serum β2-microglobulin is the single most powerful predictor of survival and can substitute for staging. Patients with β2-microglobulin levels <0.004 g/L have a median survival of 43 months, and those with levels >0.004 g/L have a survival of only 12 months. Combination of serum β2-microglobulin and albumin levels forms the basis for a three-stage International Staging System (ISS) that predicts survival. Other factors that may influence prognosis are the presence of cytogenetic abnormalities and hypodiploidy by karyotype, fluorescent in situ hybridization (FISH)–identified chromosome 17p deletion, and translocations t(4;14), (14;16), and t(14;20) (Table III-73). Chromosome 13q deletion, previously thought to predict poor outcome, is not a predictor following the use of newer agents. Microarray profiling and comparative genomic hybridization have formed the basis for RNA- and DNA-based prognostic staging systems, respectively. The ISS system, along with cytogenetic changes, is the most widely used method for assessing prognosis.
++
++
++
++
You are evaluating a 72-year-old man who has been diagnosed with monoclonal gammopathy of undetermined significance (MGUS) after finding an elevated gamma gap on routine blood testing. His bone marrow biopsy demonstrated 5% clonal plasma cells, and he has no evidence of end-organ or bone damage. He reports his appetite is excellent, and he has had no weight gain or loss in the past 1 year. His past medical history is notable for mild hypertension treated only with a diuretic and hyperlipidemia treated with atorvastatin. He has no history of latent or active tuberculosis. He still works as an international travel consultant and walks at least 2 miles three times per week. Calcium and renal function are normal on laboratory testing. His physical examination is unremarkable. Which of the following treatments is indicated at this time?
++
++
++
++
++
++
++
++
++
D. Twice-yearly plasmapheresis
++
++
E. Yearly serum protein electrophoresis, blood count, creatine, and calcium
+
++
The answer is E. (Chap. 136) Monoclonal gammopathy of undetermined significance (MGUS) is diagnosed in patients with abnormal serum protein electrophoresis, elevated serum M protein, <10% clonal plasma cells in bone marrow, no evidence of other B-cell proliferative disease, and no myeloma-related bone lesions or organ damage. No specific intervention is indicated for patients with MGUS. Follow-up once a year or less frequently is adequate except in higher risk MGUS, where serum protein electrophoresis, complete blood count, creatinine, and calcium should be repeated every 6 months. A patient with MGUS and severe polyneuropathy is considered for therapeutic intervention if a causal relationship can be assumed, especially in absence of any other potential causes for neuropathy. Therapy can include plasmapheresis and occasionally rituximab in patients with IgM MGUS or myeloma-like therapy in those with IgG or IgA disease.
++
++
++
A 64-year-old African American male is evaluated in the hospital for congestive heart failure, renal failure, and polyneuropathy. Physical examination on admission was notable for these findings and raised waxy papules in the axilla and inguinal region. Admission laboratories showed a BUN of 90 mg/dL and a creatinine of 6.3 mg/dL. Total protein was 9.0 g/dL, with an albumin of 3.2 g/dL. Hematocrit was 24%, and WBC and platelet counts were normal. Urinalysis was remarkable for 3+ proteinuria but no cellular casts. Further evaluation included an echocardiogram with a thickened left ventricle and preserved systolic function. Which of the following tests is most likely to diagnose the underlying condition?
++
++
++
++
++
B. Electromyogram (EMG) with nerve conduction studies
++
++
++
++
D. Right heart catheterization
++
++
+
++
The answer is A. (Chap. 137) This patient presents with a multisystem illness involving the heart, kidneys, and peripheral nervous system. The physical examination is suggestive of amyloidosis with classic waxy papules in the folds of his body. The laboratories are remarkable for renal failure of unclear etiology with significant proteinuria but no cellular casts. A possible etiology of the renal failure is suggested by the elevated gamma globulin fraction and low hematocrit, bringing to mind a monoclonal gammopathy perhaps leading to renal failure through amyloid AL deposition. This could also account for the enlarged heart seen on the echocardiogram and the peripheral neuropathy. The fat pad biopsy is generally reported to be 60%–80% sensitive for amyloid; however, it would not allow a diagnosis of this patient’s likely myeloma. A right heart catheterization probably would prove that the patient has restrictive cardiomyopathy secondary to amyloid deposition; however, it too would not diagnose the underlying plasma cell dyscrasia. Renal ultrasound, although warranted to rule out obstructive uropathy, would not be diagnostic. Similarly, the electromyogram and nerve conduction studies would not be diagnostic. The bone marrow biopsy is about 50%–60% sensitive for amyloid, but it would allow evaluation of the percentage of plasma cells in the bone marrow and allow the diagnosis of multiple myeloma to be made. Multiple myeloma is associated with amyloid AL in approximately 20% of cases. Light chains most commonly deposit systemically in the heart, kidneys, liver, and nervous system, causing organ dysfunction. In these organs, biopsy would show the classic eosinophilic material that, when exposed to Congo red stain, has a characteristic apple-green birefringence. Extensive multisystemic involvement typifies AL amyloidosis, and the median survival period without treatment is usually only approximately 1–2 years from the time of diagnosis.
++
++
++
You are caring for a 65-year-old African American man who was recently told by a cardiologist that he likely has heart failure due to familial amyloidosis based on echocardiography. He has a strong family history of nonsystolic heart failure including his father and a brother who both died in their 60s. Which of the following statements regarding this patient’s condition is true?
++
++
++
A. Bone marrow transplantation is curative.
++
++
B. Heart failure with diminished systolic function is more typical than heart failure with preserved systolic function.
++
++
C. The most common form involves a mutation in the transthyretin gene.
++
++
D. This disorder is more common in Hispanic Americans than African Americans.
++
++
E. Without intervention, the median survival is <2 years.
+
++
The answer is C. (Chap. 137) The most common form of familial amyloidosis is ATTRm in the updated nomenclature, caused by mutation of the abundant plasma protein transthyretin (TTR, also known as prealbumin). More than 100 TTR mutations are known, and most are associated with ATTR amyloidosis. One variant, V122I, has a carrier frequency that may be as high as 4% in the African American population and is associated with late-onset cardiac amyloidosis. The actual incidence and penetrance of disease in the African American population are the subject of ongoing research, but ATTR amyloidosis warrants consideration in the differential diagnosis of African American patients who present with concentric cardiac hypertrophy and evidence of diastolic dysfunction, particularly in the absence of a history of hypertension. DNA sequencing is the standard for diagnosis of ATTR. Without intervention, the survival period after onset of ATTR disease is 5–15 years. Standard treatment for nonsystolic heart failure is indicated. Orthotopic liver transplantation replaces the major source of variant TTR production with a source of normal TTR. Although liver transplantation can slow disease progression and improve chances of survival, it does not reverse sensorimotor neuropathy. Liver transplantations are most successful in young patients with early peripheral neuropathy; older patients with familial amyloidotic cardiomyopathy or advanced polyneuropathy often experience end-organ disease progression despite successful liver transplantation.
++
++
++
A 28-year-old man comes to your clinic with an abnormal laboratory report. He was seen in an urgent care center after a motor vehicle accident where he sustained a minor chest trauma. All studies were unremarkable, and he was discharged without any specific therapy. However, he was told that he had a low platelet count. He is asymptomatic, has no excessive bleeding, has no past medical history, takes no medications, and does not use alcohol or illicit drugs. His physical examination is totally normal. A CBC reveals normal hemoglobin and WBC with a platelet count of 80,000/μL. His peripheral blood smear is shown in Figure III-77A. Which of the following is the most likely diagnosis?
+
+
++
+
++
++
++
A. Congenital thrombocytopenia
++
++
B. Disseminated intravascular coagulopathy
++
++
C. Drug-induced thrombocytopenia
++
++
D. Idiopathic immune thrombocytopenia
++
++
E. Pseudothrombocytopenia
+
++
The answer is E. (Chap. 140) The peripheral blood smear is not consistent with true thrombocytopenia because of the clumped enlarged platelets. A repeat platelet count after the patient’s blood was collected in sodium citrate revealed a normal platelet count. Pseudothrombocytopenia is an in vitro artifact resulting from platelet agglutination via antibodies (usually IgG, but also IgM and IgA) when the calcium content is decreased by blood collection in ethylenediaminetetraacetic acid (EDTA) (the anticoagulant present in tubes [purple top] used to collect blood for complete blood counts). If a low platelet count is obtained in EDTA-anticoagulated blood, a blood smear should be evaluated and a platelet count determined in blood collected into sodium citrate (blue top tube) or heparin (green top tube), or a smear of freshly obtained unanticoagulated blood, such as from a finger stick, can be examined (Figure III-77B). The other options cause true thrombocytopenia.
+
+
++
FIGURE III-77B DIC, disseminated intravascular coagulation; RBC, red blood cell; TTP, thrombotic thrombocytopenic purpura.
+
++
++
++
A 75-year-old man is hospitalized for treatment of a deep venous thrombosis. He had recently been discharged from the hospital about 2 months ago. At that time, he had been treated for community-acquired pneumonia complicated by acute respiratory failure requiring mechanical ventilation. He was hospitalized for 21 days at that time and was discharged from a rehabilitation facility 2 weeks ago. On the day prior to admission, he developed painful swelling of his left lower extremity. A lower extremity Doppler ultrasound confirmed an occlusive thrombus of his deep femoral vein. After an initial bolus, he is started on a continuous infusion of unfractionated heparin at 1600 U/hr as he has end-stage renal disease on hemodialysis. His activated partial thromboplastin time (aPTT) is maintained in the therapeutic range. On day 5, it is noted that his platelets have fallen from 150,000/μL to 88,000/μL. What is the most appropriate action at this time?
++
++
++
A. Continue heparin infusion at the current dose and assess for anti-heparin/platelet factor 4 antibodies.
++
++
B. Stop all anticoagulation while awaiting results of anti-heparin/platelet factor 4 antibodies.
++
++
C. Stop heparin infusion and initiate argatroban.
++
++
D. Stop heparin infusion and initiate enoxaparin.
++
++
E. Stop heparin infusion and initiate lepirudin.
+
++
The answer is C. (Chap. 140) Heparin-induced thrombocytopenia (HIT) is a clinical diagnosis that must not be missed because life-threatening thrombosis can occur if not treated appropriately. The cause of HIT is the formation of antibodies to the complex of heparin and platelet factor 4 (PF4). This complex is able to activate platelets, monocytes, and endothelial cells. Many patients exposed to heparin will develop antibodies to the heparin/PF4 complex, but only a few of these will progress to develop thrombocytopenia or thrombocytopenia with thrombosis (HITT). The typical patient will develop evidence of HIT 5–14 days after exposure to heparin, although it can occur within 5 days in individuals exposed to heparin within the previous approximately 100 days, as would be expected in this patient given his recent hospitalization. The nadir platelet count is typically greater than 20,000/μL. When HIT is suspected, one should not delay treatment for laboratory testing as no currently available test has adequate sensitivity or specificity for the diagnosis. The anti-heparin/PF4 antibody test is positive in many individuals who have been exposed to heparin regardless of whether HIT is present. The platelet activation assay is more specific but less sensitive for HIT. As soon as HIT is suspected, heparin should be discontinued and replaced with an alternative form of anticoagulation to protect against development of new thromboses. Low-molecular-weight heparins (LMWH) such as enoxaparin are not appropriate treatment options in individuals with HIT. Although heparin is 10 times more likely to cause HIT, LMWHs also cause the illness and should not be used. The primary agents used for HIT in the United States are the direct thrombin inhibitors argatroban and lepirudin. Argatroban is the preferred agent for this patient because of his renal failure. The drug is not excreted by the kidneys, and no dosage adjustment is required. In contrast, lepirudin is markedly increased in renal failure, and significant dosage adjustment is required. Danaparoid has previously been used frequently for HIT/HITT, but this medication is no longer available in the United States. Other anticoagulants that are used for treatment of HITT include bivalirudin and fondaparinux, but these are not currently approved by the U.S. FDA for this indication.
++
++
++
A 48-year-old woman is evaluated by her primary care physician for a complaint of gingival bleeding and easy bruising. She has noted the problem for about 2 months. Initially, she attributed it to aspirin that she was taking intermittently for headaches, but she stopped all aspirin and nonsteroidal anti-inflammatory drug use 6 weeks ago. Her only medical history is an automobile accident 12 years previously that caused a liver laceration. It required surgical repair, and she received several transfusions of red blood cells and platelets at that time. She currently takes no prescribed medications and otherwise feels well. On physical examination, she appears well and healthy. She has no jaundice or scleral icterus. Her cardiac and pulmonary examinations are normal. The abdominal examination shows a liver span of 12 cm to percussion, and the edge is palpable 1.5 cm below the right costal margin. The spleen tip is not palpable. There are petechiae present on her extremities and hard palate with a few small ecchymoses on her extremities. A CBC shows a hemoglobin of 12.5 g/dL, hematocrit of 37.6%, WBC count of 8400/μL with a normal differential, and platelet count of 7500/μL. What tests are indicated for the workup of this patient’s thrombocytopenia?
++
++
++
A. Antiplatelet antibodies
++
++
++
++
++
++
++
++
++
++
+
++
The answer is E. (Chap. 140) This patient presents with symptoms of thrombocytopenia including bleeding gums and easy bruising. The only finding on physical examination may be petechiae at points of increased venous pressure, especially in the feet and ankles. The laboratory findings confirm thrombocytopenia but show no abnormalities in other cell lines. When evaluating isolated thrombocytopenia, one must initially consider whether there is an underlying infection or medication that is causing the platelet count to fall. There is a long list of medications that are implicated in thrombocytopenia, including aspirin, acetaminophen, penicillins, H2-blockers, heparin, and many others. This patient discontinued all medications over 6 weeks previously, and the platelet count would be expected to recover if a medication reaction were the cause. She gives no signs of any acute infection. Thus, the most likely diagnosis is immune thrombocytopenia purpura (ITP). This disorder is also known as idiopathic thrombocytopenia purpura and refers to an immune-mediated destruction of platelets and possible inhibition of platelet release from megakaryocytes. ITP can truly be idiopathic, or it can be secondary to an underlying disorder including systemic lupus erythematosus (SLE), HIV, or chronic HCV infection. The platelet count can be quite low (<5000/μL) in ITP and usually presents with mucocutaneous bleeding. Laboratory testing for ITP should include a peripheral smear that typically demonstrates large platelets with otherwise normal morphology. Initial testing should evaluate for secondary causes of ITP, including HIV antibodies, HCV antibodies, serologic testing for SLE, serum protein electrophoresis, and immunoglobulins. If anemia is also present, a direct Coombs test is indicated to assess whether there is a combined autoimmune hemolytic anemia with ITP (Evans syndrome). Antiplatelet antibody testing is not recommended as these tests have low sensitivity and specificity for ITP. In addition, bone marrow biopsy is typically not performed unless there are other abnormalities that are not explained by ITP or the patient has failed to respond to usual therapy.
++
++
++
A 54-year-old woman presents acutely with alterations in mental status and fever. She was well until 4 days previously when she began to develop complaints of myalgia and fever. Her symptoms progressed rapidly, and today her husband noted her to be lethargic and unresponsive when he awakened. She has recently felt well otherwise. Her only current medication is atenolol 25 mg daily for hypertension. On physical examination, she is responsive only to sternal rub and does not vocalize. Her vital signs are as follows: blood pressure 165/92 mmHg, heart rate 114 bpm, temperature 38.7°C (101.7°F), respiratory rate 26 breaths/min, and oxygen saturation 92% on room air. Her cardiac examination shows a regular tachycardia. Her lungs have bibasilar crackles. The abdominal examination is unremarkable. No hepatosplenomegaly is present. There are petechiae on the lower extremities. Her CBC shows a hemoglobin of 8.8 g/dL, hematocrit of 26.4%, WBC count of 10.2/μL (89% polymorphonuclear cells, 10% lymphocytes, 1% monocytes), and a platelet count of 54,000/μL. A peripheral blood smear is shown in Figure III-80. Her basic metabolic panel shows a sodium of 137 mEq/L, potassium of 5.4 mEq/L, chloride of 98 mEq/L, bicarbonate of 18 mEq/L, BUN of 89 mg/dL, and creatinine of 2.9 mg/dL. Which of the following most correctly describes the pathogenesis of the patient’s condition?
+
+
++
+
++
++
++
A. Development of autoantibodies to a metalloproteinase that cleaves von Willebrand factor
++
++
B. Development of autoantibodies to the heparin/platelet factor 4 complex
++
++
C. Direct endothelial toxicity initiated by an infectious agent
++
++
D. Inherited disorder of platelet granule formation
++
++
E. Inherited disorder of von Willebrand factor that precludes binding with factor VIII
+
++
The answer is A. (Chap. 140) This patient presents with the classic pentad of thrombotic thrombocytopenic purpura (TTP): fever, neurologic symptoms, acute renal failure, thrombocytopenia, and microscopic angiopathic hemolytic anemia (MAHA). The peripheral blood smear shows schistocytes and decreased platelets consistent with MAHA. Although this is the classic presentation of TTP, it is not necessary to have all five characteristics for an individual to be diagnosed with TTP. In recent years, the pathogenesis of inherited and idiopathic TTP has been discovered to be due to a deficiency of antibodies directed against the ADAMTS13 protein. The ADAMTS13 protein is a metalloproteinase that cleaves von Willebrand factor (vWF). In the absence of ADAMTS13, ultra-large vWF multimers circulate in the blood and can cause pathogenic platelet adhesion and activation, resulting in microvascular ischemia and microangiopathic hemolytic anemia. However, it appears as if there is a necessary inciting event because not all individuals with an inherited deficiency of ADAMTS13 develop TTP. Some drugs have been implicated as causative agents in TTP. Ticlopidine and possibly clopidogrel cause TTP by inducing antibody formation. Other drugs including mitomycin C, cyclosporine, and quinine can cause TTP by causing direct endothelial toxicity.
++
In patients presenting with new thrombocytopenia, with or without evidence of renal insufficiency and other elements of classic TTP, laboratory data should be obtained to rule out DIC and to evaluate for evidence of MAHA. Findings to support the TTP diagnosis include an increased lactate dehydrogenase and indirect bilirubin, decreased haptoglobin, and increased reticulocyte count, with a negative direct antiglobulin test. The peripheral smear should be examined for evidence of schistocytes. Polychromasia is usually also present due to the increased number of young RBCs, and nucleated RBCs are often present, which is thought to be due to infarction in the microcirculatory system of the bone marrow. A diagnosis of TTP can be made based on clinical factors. It should be differentiated from DIC, which causes MAHA but has a predominant coagulopathy. Hemolytic-uremic syndrome also causes MAHA and appears very similar to TTP in clinical presentation, although neurologic symptoms are less prominent. Often a preceding diarrheal illness alerts one to hemolytic-uremic syndrome as the cause of MAHA. It is important to make a prompt and correct diagnosis as the mortality of TTP without treatment is 85%–100%, decreasing to 10%–30% with treatment. The primary treatment for TTP remains plasma exchange. Plasma exchange should be continued until the platelet count returns to the normal range and there is no further evidence of hemolysis for at least 2 days. Glucocorticoids can be used as adjunctive treatment in TTP but are not effective as the sole therapy. Additionally, other immunomodulatory therapies have been reported to be successful in refractory or relapsing TTP, including rituximab, vincristine, cyclophosphamide, and splenectomy. A significant relapse rate is noted; 25%–45% of patients relapse within 30 days of initial “remission,” and 12%–40% of patients have late relapses. Relapses are more frequent in patients with severe ADAMTS13 deficiency at presentation.
++
++
++
A 54-year-old woman presents acutely with alterations in mental status and fever. She was well until 4 days previously when she began to develop complaints of myalgia and fever. Her symptoms progressed rapidly, and today her husband noted her to be lethargic and unresponsive when he awakened. She has recently felt well otherwise. Her only current medication is atenolol 25 mg daily for hypertension. On physical examination, she is responsive only to sternal rub and does not vocalize. Her vital signs are as follows: blood pressure 165/92 mmHg, heart rate 114 bpm, temperature 38.7°C (101.7°F), respiratory rate 26 breaths/min, and oxygen saturation 92% on room air. Her cardiac examination shows a regular tachycardia. Her lungs have bibasilar crackles. The abdominal examination is unremarkable. No hepatosplenomegaly is present. There are petechiae on the lower extremities. Her CBC shows a hemoglobin of 8.8 g/dL, hematocrit of 26.4%, WBC count of 10.2/μL (89% polymorphonuclear cells, 10% lymphocytes, 1% monocytes), and a platelet count of 54,000/μL. A peripheral blood smear is shown in Figure III-80. Her basic metabolic panel shows a sodium of 137 mEq/L, potassium of 5.4 mEq/L, chloride of 98 mEq/L, bicarbonate of 18 mEq/L, BUN of 89 mg/dL, and creatinine of 2.9 mg/dL. What is the best initial treatment for the patient?
+
+
++
+
++
++
++
A. Acyclovir 10 mg/kg intravenously every 8 hours
++
++
B. Ceftriaxone 2 g intravenously daily plus vancomycin 1 g intravenously twice daily
++
++
++
++
D. Methylprednisolone 1 g intravenously
++
++
+
++
The answer is E. (Chap. 140) This patient presents with the classic pentad of thrombotic thrombocytopenic purpura (TTP): fever, neurologic symptoms, acute renal failure, thrombocytopenia, and microscopic angiopathic hemolytic anemia (MAHA). The peripheral blood smear shows schistocytes and decreased platelets consistent with MAHA. Although this is the classic presentation of TTP, it is not necessary to have all five characteristics for an individual to be diagnosed with TTP. In recent years, the pathogenesis of inherited and idiopathic TTP has been discovered to be due to a deficiency of antibodies directed against the ADAMTS13 protein. The ADAMTS13 protein is a metalloproteinase that cleaves von Willebrand factor (vWF). In the absence of ADAMTS13, ultra-large vWF multimers circulate in the blood and can cause pathogenic platelet adhesion and activation, resulting in microvascular ischemia and microangiopathic hemolytic anemia. However, it appears as if there is a necessary inciting event because not all individuals with an inherited deficiency of ADAMTS13 develop TTP. Some drugs have been implicated as causative agents in TTP. Ticlopidine and possibly clopidogrel cause TTP by inducing antibody formation. Other drugs including mitomycin C, cyclosporine, and quinine can cause TTP by causing direct endothelial toxicity.
++
In patients presenting with new thrombocytopenia, with or without evidence of renal insufficiency and other elements of classic TTP, laboratory data should be obtained to rule out DIC and to evaluate for evidence of MAHA. Findings to support the TTP diagnosis include an increased lactate dehydrogenase and indirect bilirubin, decreased haptoglobin, and increased reticulocyte count, with a negative direct antiglobulin test. The peripheral smear should be examined for evidence of schistocytes. Polychromasia is usually also present due to the increased number of young RBCs, and nucleated RBCs are often present, which is thought to be due to infarction in the microcirculatory system of the bone marrow. A diagnosis of TTP can be made based on clinical factors. It should be differentiated from DIC, which causes MAHA but has a predominant coagulopathy. Hemolytic-uremic syndrome also causes MAHA and appears very similar to TTP in clinical presentation, although neurologic symptoms are less prominent. Often a preceding diarrheal illness alerts one to hemolytic-uremic syndrome as the cause of MAHA. It is important to make a prompt and correct diagnosis as the mortality of TTP without treatment is 85%–100%, decreasing to 10%–30% with treatment. The primary treatment for TTP remains plasma exchange. Plasma exchange should be continued until the platelet count returns to the normal range and there is no further evidence of hemolysis for at least 2 days. Glucocorticoids can be used as adjunctive treatment in TTP but are not effective as the sole therapy. Additionally, other immunomodulatory therapies have been reported to be successful in refractory or relapsing TTP, including rituximab, vincristine, cyclophosphamide, and splenectomy. A significant relapse rate is noted; 25%–45% of patients relapse within 30 days of initial “remission,” and 12%–40% of patients have late relapses. Relapses are more frequent in patients with severe ADAMTS13 deficiency at presentation.
++
++
++
All of the following genetic mutations are associated with an increased risk of deep venous thrombosis EXCEPT:
++
++
++
A. Factor V Leiden mutation
++
++
B. Glycoprotein 1b platelet receptor
++
++
C. Heterozygous protein C deficiency
++
++
++
++
E. Tissue plasminogen activator
+
++
The answer is B. (Chap. 142) Venous thrombosis occurs through activation of the coagulation cascade primarily through the exposure to tissue factor, and the genetic factors that contribute to a predisposition to venous thrombosis typically are those polymorphisms affecting procoagulant or fibrinolytic pathways. In contrast, arterial thrombosis occurs in the setting of a platelet activation, and the genetic predisposition for arterial thrombosis includes mutations that affect platelet receptors or redox enzymes. The most common inherited risk factors for venous thrombosis are the factor V Leiden mutation and the prothrombin 20210 mutation. Other mutations predisposing an individual to venous thrombosis include inherited deficiency of protein C or S and mutations of fibrinogen, tissue plasminogen activator, thrombomodulin, or plasminogen activator inhibitor. The glycoprotein Ib platelet receptor mutation would increase the risk of arterial, but not venous, thrombosis.
++
++
++
A 76-year-old man presents to an urgent care clinic with pain in his left leg for 4 days. He also describes swelling in his left ankle, which has made it difficult for him to ambulate. He is an active smoker and has a medical history remarkable for gastroesophageal reflux disease, prior deep venous thrombosis (DVT) 9 months ago that resolved, and well-controlled hypertension. Physical examination is revealing for 2+ edema in his left ankle. A D-dimer is ordered and is elevated. Which of the following makes D-dimer less predictive of DVT in this patient?
++
++
++
++
++
B. History of active tobacco use
++
++
C. Lack of suggestive clinical symptoms
++
++
D. Negative Homan sign on examination
++
++
E. Previous DVT in the past year
+
++
The answer is A. (Chap. 142) D-dimer is a degradation product of cross-linked fibrin and is elevated in conditions of ongoing thrombosis. Low concentrations of D-dimer are considered to indicate the absence of thrombosis. Patients over the age of 70 will frequently have elevated D-dimers in the absence of thrombosis, making this test less predictive of acute disease. Clinical symptoms are often not present in patients with deep venous thrombosis (DVT) and do not affect interpretation of a D-dimer. Tobacco use, while frequently considered a risk factor for DVT, and previous DVT should not affect the predictive value of D-dimer. Homan sign, calf pain elicited by dorsiflexion of the foot, is not predictive of DVT and is unrelated to D-dimer.
++
++
++
A 22-year-old woman comes to the emergency department complaining of 12 hours of shortness of breath. The symptoms began toward the end of a long car ride home from college. She has no past medical history, and her only medication is an oral contraceptive. She smokes occasionally, but the frequency has increased recently because of midterm examinations. On physical examination, she is afebrile with a respiratory rate of 22 breaths/min, blood pressure of 120/80 mmHg, heart rate of 110 bpm, and oxygen saturation of 92% (room air). The rest of her physical examination is normal. A chest radiograph and CBC are normal. Her serum pregnancy test is negative. Which of the following is the indicated management strategy?
++
++
++
A. Check D-dimer and, if normal, discharge with nonsteroidal anti-inflammatory therapy.
++
++
B. Check D-dimer and, if normal, obtain lower extremity ultrasound.
++
++
C. Check D-dimer and, if abnormal, treat for DVT/pulmonary embolism (PE).
++
++
D. Check D-dimer and, if abnormal, obtain contrast multislice CT of chest.
++
++
E. Obtain contrast multislice CT of chest.
+
++
The answer is E. (Chaps. 142 and 300) The clinical probability of pulmonary embolism (PE) can be delineated into likely versus unlikely using the clinical decision rule shown in Table III-84. In those with a score ≤4, PE is unlikely and a D-dimer test should be performed. A normal D-dimer combined with an unlikely clinical probability of PE identifies patients who do not need further testing or anticoagulation therapy. Those with either a likely clinical probability (score >4) or an abnormal D-dimer (with unlikely clinical probability) require an imaging test to rule out PE. Currently the most attractive imaging method to detect PE is the multislice CT scan. It is accurate and, if normal, safely rules out PE. This patient has a clinical probability score of 4.5 because of her resting tachycardia and the lack of an alternative diagnosis at least as likely as PE. Therefore, there is no indication for measuring D-dimer, and she should proceed directly to multislice CT of the chest. If this cannot be performed expeditiously, she should receive one dose of LMWH while awaiting the test.
++